Neurology Flashcards

1
Q

What is a cluster headache?

A

Most disabling of primary headache disorders

How well did you know this?
1
Not at all
2
3
4
5
Perfectly
2
Q

How common is a cluster headache?

A
Distinct from migraine
Much rarer than migraine 1 per 1000
More common in men
Affects adults typically between 20-40
Commoner in smokers
How well did you know this?
1
Not at all
2
3
4
5
Perfectly
3
Q

What can increase your risk of getting cluster headaches?

A

Smoker
Male
Autosomal dominant gene has role

How well did you know this?
1
Not at all
2
3
4
5
Perfectly
4
Q

What is the pathophysiology of cluster headaches?

A

Unknown
May be due to superficial temporal artery smooth muscle hyper-reactivity to serotonin
Hypothalamic grey matter abnormalities

How well did you know this?
1
Not at all
2
3
4
5
Perfectly
5
Q

How do cluster headaches present?

A

Abrupt onset
Excruciating pain around one eye, temple or forehead
Ipsilateral cranial autonomic features
- Eye may become watery and bloodshot with lid swelling, lacrimation
- Facial flushing
- Rhinorrhea
- Miosis +/- ptosis (20%)
Pain unilateral and almost always affects the same side
Rises to crescendo over minutes and lasts 15-160 mins, once or twice a day - usually at same time
Often nocturnal/early morning - often wakes from sleep
+/- vomiting
Episodic - clusters last 4-12 weeks and are followed by pain free periods of months or even 1-2 years before next cluster
Can be chronic

How well did you know this?
1
Not at all
2
3
4
5
Perfectly
6
Q

What could be a differential for cluster headaches?

A

Migraine

How well did you know this?
1
Not at all
2
3
4
5
Perfectly
7
Q

How are cluster headaches diagnosed?

A

Clinical diagnosis

At least 5 headache attacks fulfilling above criteria

How well did you know this?
1
Not at all
2
3
4
5
Perfectly
8
Q

How are acute attacks of cluster headaches treated?

A

Analgesics are unhelpful
100% 15L for 15 mins via non-rebreathable mask
Triptan (selective serotonin agonist) - reduces vascular inflammation

How well did you know this?
1
Not at all
2
3
4
5
Perfectly
9
Q

How are cluster headaches prevented?

A

CCB eg verapamil is first line prophylaxis
Avoid alcohol during cluster period
Corticosteroids eg prednisolone may help during cluster

How well did you know this?
1
Not at all
2
3
4
5
Perfectly
10
Q

How common are headaches?

A

One of the most common symptoms
Symptoms are unpleasant, disabling and common worldwide and have substantial economic impact because of time lost from work

How well did you know this?
1
Not at all
2
3
4
5
Perfectly
11
Q

What are the different types of headache?

A

Primary - no underlying cause relevant to headache
Secondary - underlying cause, need to identify underlying cause eg giant cell arteritis
Other - trigeminal neuralgia (facial pain) - painful cranial neuropathy

How well did you know this?
1
Not at all
2
3
4
5
Perfectly
12
Q

Name an example of a primary headache

A

Migraine (20% population)
Cluster
Tension (affect 99% in lifetime)

How well did you know this?
1
Not at all
2
3
4
5
Perfectly
13
Q

What are the red flags associated with headaches?

A
HIV/immunosuppressed
Fever
Thunderclap headache (subarachnoid haemorrhage)
Seizure and new headache
Suspected meningitis
Suspected encephalitis
Red eye? acute glaucoma
Headache + new focal neurology eg papilloedema
How well did you know this?
1
Not at all
2
3
4
5
Perfectly
14
Q

Name an example of a secondary headache cause

A

Meningitis, subarachnoid haemorrhage, giant cell arteritis, medication overuse headache

How well did you know this?
1
Not at all
2
3
4
5
Perfectly
15
Q

What is a migraine?

A

Recurrent throbbing headache often preceded by an aura and associated with nausea, vomiting and visual changes
Migraine aura may affect the patients eyesight with visual phenomena such as fortification spectra (zig-zag lines), shimmering or scotomas (black holes in visual field), but may also result in pins and needles, dysphasia and rarely weakness of limbs and motor function

How well did you know this?
1
Not at all
2
3
4
5
Perfectly
16
Q

How common are migraines?

A

Most common cause of episodic headache (recurrent)
More common in women
In 90% onset before 40
If onset > 50 then pathology sought
Usually severity of migraine decreases with advancing age

How well did you know this?
1
Not at all
2
3
4
5
Perfectly
17
Q

What can cause migraines?

A

No specific causes but partial triggers
CHOCOLATE - Chocolate, Hangovers, Orgasms, Cheese, Oral contraceptives, Lie-ins, Alcohol, Tumult (loud noise), Exercise
Brain chemical imbalance may be cause
May be caused by changes in brainstem and its interactions with trigeminal nerve

How well did you know this?
1
Not at all
2
3
4
5
Perfectly
18
Q

What can increase your risk of getting migraines?

A

Strong genetic component - FHx
Female
Age - can occur at any age but majority have first migraine in adolescence

How well did you know this?
1
Not at all
2
3
4
5
Perfectly
19
Q

What is the pathophysiology of migraine?

A

Genetic factors play role in neuronal hyper-excitability
Changes in brainstem blood flow lead to an unstable trigeminal nerve nucleus and nuclei in basal thalamus
Cortical spreading depression - self-propagating wave of neuronal and glial depolarisation that spreads across the cerebral cortex thought to cause aura of migraine and leads to release of inflammatory mediators which impact on trigeminal nerve nucleus
Results in release of vasoactive neuropeptides including calcitonin-gene-related peptide and substance P - results in neurogenic inflammation - vasodilation and plasma protein extravasation leading to pain that propagates all over cerebral cortex

How well did you know this?
1
Not at all
2
3
4
5
Perfectly
20
Q

How do migraines present?

A

Prodrome that precedes headache by hours/days consisting of - yawning, cravings, mood/sleep changes
Migraine without aura
- Attacks last 4-72 hours
- Two of following - unilateral, pulsing, moderate/severe head pain, aggravated by routine physical activity
- During headache at least one of - nausea and/or vomiting, photophobia, phonophobia
- Not attributed to another disorder
Migraine with aura
- At least 2 attacks
- Aura precedes attack by minutes and may persist during it
- Visual - chaotic cascading, jumbling, distorting lines, dots or zigzags, scotoma, hemianopia
- Somatosensory - paraesthesiae spreading from fingers to toes
- Unilateral, pulsatile headache
General
- At least 2 of - unilateral pain, throbbing eye pain, moderate-severe intensity, motion sensitivity
- At least one of - nausea/vomiting, photophobia/phonophobia, normal examination with no attributable cause

How well did you know this?
1
Not at all
2
3
4
5
Perfectly
21
Q

Give 5 possible differential diagnoses of migraine?

A

Tension headache, cluster headache, medication over-use headache
Sudden migraine may resemble meningitis or subarachnoid haemorrhage
Visual and hemisensory symptoms must be distinguished from thromboembolic TIAs
Brain tumour and temporal arteritis

How well did you know this?
1
Not at all
2
3
4
5
Perfectly
22
Q

How is migraine diagnosed?

A
Mainly clinical diagnosis
Always examine
- Eyes - for papilloedema 
- BP
- Head and neck 
Exclude other causes
- CRP/ESR
- Red flags - indication for neuroimaging
- Lumbar puncture
How well did you know this?
1
Not at all
2
3
4
5
Perfectly
23
Q

What are the indications for lumbar puncture?

A

Worst headache of life - thunderclap
Severe rapid onset headache, progressive headaches, unresponsive headaches
Neuroimaging should precede lumbar puncture to rule out mass/lesion/raised ICP

How well did you know this?
1
Not at all
2
3
4
5
Perfectly
24
Q

What are the indications for neuroimaging?

A
Worst/severe headache
Change in pattern of migraine
Abnormal neurological exam
Onset > 50
Epilepsy
Posteriorly located headache
How well did you know this?
1
Not at all
2
3
4
5
Perfectly
25
Q

How do you treat migraines?

A
Reduce triggers 
Acute
- Triptans eg sumatriptan
- NSAIDs 
- +/- anti-emetic
How well did you know this?
1
Not at all
2
3
4
5
Perfectly
26
Q

How do triptans work?

A

Selectively stimulate 5-hydroxytryptamine receptors in brain
Contraindicated in IHD, coronary spasm and uncontrolled high BP
S/E arrhythmias or angina +/- MI

How well did you know this?
1
Not at all
2
3
4
5
Perfectly
27
Q

Why do you prescribe NSAIDs for migraines?

A

Less likely to get medication over-use headache

Avoid paracetamol or ibuprofen

How well did you know this?
1
Not at all
2
3
4
5
Perfectly
28
Q

How do you prevent migraines?

A

If more than 2 attacks a month, or acute treatment required more than 2x a week
Beta-blocker
Tricyclic anti-depressant
Anti-convulsant

How well did you know this?
1
Not at all
2
3
4
5
Perfectly
29
Q

What is a tension headache?

A

Most chronic daily and recurrent headaches are tension headaches

How well did you know this?
1
Not at all
2
3
4
5
Perfectly
30
Q

How common are tension headaches?

A

Commonest primary headache
Can be episodic < 15 days/month or chronic > 15 days/month for at least 3 months
No organic cause

How well did you know this?
1
Not at all
2
3
4
5
Perfectly
31
Q

What can cause tension headaches?

A
Stress
Sleep deprivation
Bad posture
Hunger
Eyestrain
Anxiety
Noise
How well did you know this?
1
Not at all
2
3
4
5
Perfectly
32
Q

How do tension headaches present?

A

Usually one of following
- Bilateral, pressing/tight non-pulsatile, mild/moderate intensity, +/- scalp tenderness
Without vomiting or sensitivity to head movement, no aura
Not aggravated by physical acitivty
Tight band-like sensation
Pressure behind eyes, mild-moderate pain
Headaches can last from 30 mins to 7 days
Not attributed to another disorder

How well did you know this?
1
Not at all
2
3
4
5
Perfectly
33
Q

What could be a differential diagnosis of tension headaches?

A

Migraine
Cluster headache
Giant cell arteritis
Drug-induced headache

How well did you know this?
1
Not at all
2
3
4
5
Perfectly
34
Q

How are tension headaches diagnosed?

A

Clinical diagnosis from history

How well did you know this?
1
Not at all
2
3
4
5
Perfectly
35
Q

How are tension headaches treated?

A

Reassurance and lifestyle advice - regular exercise, avoid triggers
Stress relief
Symptomatic treatment of episodes occuring > 2 days a week
- Aspirin, paracetamol, NSAIDs, no opioids
Limit analgesia to no more than 6 days a month to reduce chance of medication over use headaches
Tricyclic antidepressants

How well did you know this?
1
Not at all
2
3
4
5
Perfectly
36
Q

How do medication overuse headaches present?

A

Worsens whilst on regular analgesia especially opioids
Other causes mixed analgesics eg co-codamol, ergotamine, triptans
Common reason for episodic headache becoming chronic daily headache

How well did you know this?
1
Not at all
2
3
4
5
Perfectly
37
Q

What is a TIA?

A

Brief episode of neurological dysfunction due to temporary focal cerebral ischaemia without infarction
Symptoms generally resolve within 24 hours

How well did you know this?
1
Not at all
2
3
4
5
Perfectly
38
Q

How common are TIAs?

A

15% first strokes preceded by TIA, also foreshadowing of MI
More common in men
Black ethnicity at greater risk due to hypertension and atherosclerosis predisposition

How well did you know this?
1
Not at all
2
3
4
5
Perfectly
39
Q

What causes TIA?

A

Small vessel occlusion
Atherothromboembolism from carotid chief cause
Cardioembolism resulting from microemboli from
- Mural thrombus post-MI or in AF
- Valve disease
- Prosthetic valve
Hyperviscosity
- Polycythaemia, sickle cell anaemia, extremely raised WCC or myeloma
- Can result from hypoperfusion - most important to consider in young people - cardiac dysrhythmia, postural hypotension, decreased flow through atheromatous arteries

How well did you know this?
1
Not at all
2
3
4
5
Perfectly
40
Q

What can increase your risk of a TIA?

A
Increasing age
Hypertension
Smoking
Diabetes
Heart disease - valvular, ischaemic, or AF
Past TIA
Raised packed cell volume
Peripheral arterial disease
Polycythaemia vera
Combined OC
Hyperlipidaemia
Excess alcohol
Clotting disorder
Vasculitis eg SLE, giant cell arteritis, rare risk factor
How well did you know this?
1
Not at all
2
3
4
5
Perfectly
41
Q

What happens to the brain during a TIA?

A

Cerebral ischaemia resulting in lack of O2 and nutrients to brain result in cerebral dysfunction
Short lived period of ischaemia with symptoms only lasting a maximum of 5-15 minutes after onset and then resolves before irreversible cell death occurs
Symptoms gradually progressing suggest different pathology eg demyelination, tumour or migraine

How well did you know this?
1
Not at all
2
3
4
5
Perfectly
42
Q

How does a TIA present?

A

Sudden loss of function, usually only lasts for minutes
Complete recovery and no evidence of infarction on imaging
Site of TIA often suggested by symptoms

How well did you know this?
1
Not at all
2
3
4
5
Perfectly
43
Q

How does an anterior circulation TIA present?

A

90%
Weak, numb contralateral leg +/- similar if milder arm
Hemiparesis - weakness of entire side of body
Hemisensory disturbance
Dysphasia
Amaurosis fugaux- sudden transient loss of vision in one eye, temporary reduction in retinal, ophthalmic or ciliary blood flow leading to temporary retinal hypoxia, TIA causing this often first clinical evidence of ICA stenosis

How well did you know this?
1
Not at all
2
3
4
5
Perfectly
44
Q

How does a posterior circulation TIA present?

A
10%
Diplopia
Vertigo
Chocking and dysarthria
Ataxia
Hemisensory loss
Hemianopia vision loss
Transient global amnesia 
Tetraparesis
How well did you know this?
1
Not at all
2
3
4
5
Perfectly
45
Q

What is a possible differential diagnosis of a TIA?

A

Cannot differentiate from stroke until full recovery
Hypoglycaemia, migraine aura, focal epilepsy
Intracranial lesion - tumour or subdural haematoma
Syncope due to arrhythmia
Todd’ paralysis - transient weakness of arm, hand or leg after seizure
Retinal or vitreous haemorrhage
Giant cell arteritis

How well did you know this?
1
Not at all
2
3
4
5
Perfectly
46
Q

How is a TIA diagnosed?

A
Often solely on Hx
Bloods
- FBC - polycythaemia?
- ESR - raised in vasculitis
- Glucose - hypoglycaemia?
- Creatinine, electrolytes
- Cholesterol
Carotid artery doppler USS to look for stenosis/atheroma
MR/CT angiography if stenosis to determine extent
ECG - AF/MI ischaemia
CT/diffusion weight MRI
Echo/cardiac monitoring to assess for cardiac cause
How well did you know this?
1
Not at all
2
3
4
5
Perfectly
47
Q

What is the ABCD2 score?

A
To assess risk of stroke after TIA
- Age > 60 = 1
- BP > 140/90mmHg > 1
- Clinical features
  - Unilateral weakness = 2
  - Speech disturbance w/o weakness = 1
- Duration of symptoms
  - Lasting more than 1 hour = 2
  - Lasting 10-59 minutes = 1
- Diabetes = 1
Score greater than 6 strongly predicts stroke and referred to specialise immediately
Score greater than 4 assessed by specialist with 24 hours
All patients with suspected TIA seen within 7 days
How well did you know this?
1
Not at all
2
3
4
5
Perfectly
48
Q

When are you also at an increased risk of stroke?

A

AF
More than 1 TIA in one week
TIA whilst on anti-coagulant

How well did you know this?
1
Not at all
2
3
4
5
Perfectly
49
Q

How do you treat TIAs?

A

Antiplatelet drugs
- Aspirin immediatley + dipyridamole for 2 weeks then lower dose
- P2Y12 inhibitor long term eg clopidogrel
Anticoagulant in AF, mitral stenosis or recent big septal MI eg warfarin
Long term statins
Control CVS risk factors - antihypertensives, improve diet, stop smoking
Don’t drive for at least 4 weeks following TIA

How well did you know this?
1
Not at all
2
3
4
5
Perfectly
50
Q

What is a stroke?

A

Syndrome of rapid onset neurological deficit caused by focal, cerebral, spinal or retinal infarction
Characterised by rapidly developing signs of focal or global disturbance of cerebral functions, lasting for more than 24 hours or leading to death

How well did you know this?
1
Not at all
2
3
4
5
Perfectly
51
Q

How common are strokes?

A

Major neurological disease of our time
3rd most common cause of death in high income countries - 11% all deaths in UK
Leading cause of adult disability worldwide
Higher rates in Asian and black African populations than Caucasian
Uncommon under 40
Incidence increases with age
More common in men
Incidence falling due to more vigorous approach to risk factors in primary care ie statin use and control of BP
Ischaemia/infarction 80%
Haemorrhagic 17%
Other causes 3%

How well did you know this?
1
Not at all
2
3
4
5
Perfectly
52
Q

What can cause ischaemic strokes?

A

Small vessel occlusion/thrombosis in situ
Cardiac emboli from AF, MI or IE
Large artery stenosis
Atherothromboembolism
Hypoperfusion, vasculitis, hyperviscosity
Hypoperfusion - watershed stroke where sudden BP drop by more than 40mmHg, then low cerebral blood flow leading to global ischaemia and watershed infarcts in vulnerable areas of cortex between boundaries of different arterial territories

How well did you know this?
1
Not at all
2
3
4
5
Perfectly
53
Q

What can cause haemorrhagic strokes?

A

CNS bleeds due to trauma, aneurysm rupture, anti-coagulation, thrombolysis, carotid artery dissection, SAH

How well did you know this?
1
Not at all
2
3
4
5
Perfectly
54
Q

What can cause strokes in young people?

A

Vasculitis
Thrombophilia
SAH
Carotid artery dissection - spontaneous/neck trauma
Venous sinus thrombosis - very rare, may occur in pregnancy, hypercoagulable states and thrombotic disorders or with dehydration or malignancy
- Leads to cortical infarction, seizures, raised ICP

How well did you know this?
1
Not at all
2
3
4
5
Perfectly
55
Q

What can increase your risk of a stroke?

A
Male
Black/Asian
Hypertension
Past TIA
Smoking
DM
Increasing age
Heart disease (valvular/ischaemic)
Alcohol
Polycythaemia, thrombophilia
AF 
Hypercholesterolaemia 
Combined OC
Vasculitis
IE
How well did you know this?
1
Not at all
2
3
4
5
Perfectly
56
Q

What occurs in ischaemic strokes?

A

Arterial disease and atherosclerosis
Thrombosis occurs at site of atheromatous plaque in carotid/vertebral/cerebral arteries
Large artery stenosis acts as embolism source rather than occluding vessel hypertension results in small infarcts known as lacunes and/or gradual accumulation of diffuse ischaemic change in deep white matter

How well did you know this?
1
Not at all
2
3
4
5
Perfectly
57
Q

What can cause haemorrhagic strokes?

A

Hypertension resulting in micro-aneurysm rupture
Cerebral amyloid angiopathy - deposition of amyloid B in walls of small and medium sized arteries in normotensive patients
Space occupying lesions
Young adults 1/5 strokes due to carotid/vertebral artery dissection

How well did you know this?
1
Not at all
2
3
4
5
Perfectly
58
Q

How do ACA strokes present?

A

Leg weakness and sensory disturbance
Gait apraxia - difficulty walking
Truncal ataxia
Incontinence
Drowsiness - consciousness controlled partly by frontal lobe
Akinetic mutism - decrease in spontaneous speech, stuporous state

How well did you know this?
1
Not at all
2
3
4
5
Perfectly
59
Q

How do MCA strokes present?

A

Contralateral arm and leg weakness and sensory loss
Hemianopia
Aphasia and dysphasia (if in dominant hemisphere)
Facial droop

How well did you know this?
1
Not at all
2
3
4
5
Perfectly
60
Q

How do PCA strokes present?

A

Contralateral homonymous hemianopia
Cortical blindness
Visual agnosia - cannot interpret visual info
Prosopagnosia - cannot detect faces
Colour naming and discriminate problems
Unilateral headache - rare in ischaemic stroke

How well did you know this?
1
Not at all
2
3
4
5
Perfectly
61
Q

How do posterior circulation strokes present?

A

More catastrophic as wide region supplied
Likely to get locked in
Motor deficits such as hemiparesis or tetraparesis and facial paralysis
Dysarthria and speech impairment
Vertigo, nausea and vomiting
Visual disturbances
Altered consciousness

How well did you know this?
1
Not at all
2
3
4
5
Perfectly
62
Q

What is a lacunar stroke?

A

Small cortical stroke in midbrain, internal capsule presenting with one of

  • Unilateral weakness and/or sensory deficit of face and arm, arm and leg or all three
  • Pure sensory loss
  • Ataxic hemiparesis (cerebellar or motor symptoms)
How well did you know this?
1
Not at all
2
3
4
5
Perfectly
63
Q

How do you distinguish between a haemorrhagic/ischaemic infarct?

A

No reliable way of distinguishing between them
Intracerebral haemorrhage associated with severe headache/coma (signs of raised ICP)
Patients on oral anticoagulants assumed to have haemorrhagic unless proved otherwise

How well did you know this?
1
Not at all
2
3
4
5
Perfectly
64
Q

What is a possible differential diagnosis of stroke?

A

Hypoglycaemia
Migraine aura
Focal epilepsy
Syncope due to arrhythmia

How well did you know this?
1
Not at all
2
3
4
5
Perfectly
65
Q

How are stroke diagnosed?

A

Urgent CT head/MRI before treatment
- Rules out haemorrhagic stroke before beginning thrombolysis
- Infarction seen as low density lesion - subtle changes evident within 3 hours
- In MRI, hyper-intense within hours of onset
Pulse, BP, ECG
- Look for AF
- Careful treating BP as may compromise cerebral perfusion
Bloods
- FBC - looking for thrombocytopenia and polycythaemia
- Blood glucose - ruling out hypoglycaemia

How well did you know this?
1
Not at all
2
3
4
5
Perfectly
66
Q

How is stroke treated?

A

Maximise reversible ischaemic tissue
- Ensure dehydration
- Keep O2 sats > 95%
- If ischaemia stroke confirmed by CT then thrombolysis
Thrombolysis
- Can be give up to 4.5 hours post symptom onset
- Rule out haemorrhage first
- Tissue plasminogen activator IV alteplase
- Start antiplatelet therapy clopidogrel 24 hours after thrombolysis
If time of onset unknown then aspirin daily for 2 weeks then lifelong clopidogrel
Haemorrhagic
- Frequent GCS monitoring
- Antiplatelets CI
- Any anti-coagulants reversed for warfarin reversal used beriplex and vit K
- Control hypertension
- Manual decompression of raised ICP/diuretics
- Surgery may be required

How well did you know this?
1
Not at all
2
3
4
5
Perfectly
67
Q

When can you not give thrombolysis?

A
Recent surgery in last 3 months
Recent arterial puncture
Hx of active malignancy
Evidence of brain aneurysm
Patient on anti-coagulation
Severe liver disease
Acute pancreatitis
Clotting disorder
How well did you know this?
1
Not at all
2
3
4
5
Perfectly
68
Q

How do you prevent a stroke?

A

Platelet treatment (lifelong) eg aspirin + dipyridamole/clopidogrel
Cholesterol treatment like statins
AF treatment - warfarin or NOACs eg pixiban
BP treatment

How well did you know this?
1
Not at all
2
3
4
5
Perfectly
69
Q

What is a subarachnoid haemorrhage?

A

Spontaneous bleeding into subarachnoid space - between arachnoid layer of meninges and pia mater
Can often be catastrophic

How well did you know this?
1
Not at all
2
3
4
5
Perfectly
70
Q

How common are subarachnoid haemorrhages?

A

Typical age 35-65
Account for 5% of strokes
Most common cause is rupture of Berry aneurysms

How well did you know this?
1
Not at all
2
3
4
5
Perfectly
71
Q

What can cause subarachnoid haemorrhages?

A
Rupture of berry aneurysms (80%)
- Rupture of junction of posterior communicating artery with internal carotid or anterior communicating artery with anterior cerebral artery (in circle of Willis)
- 15% multiple
- Associated with PKD, and coarctation of aorta
Arteriovenous malformation (10%)
- Vascular developmental malformation often with fistula between arterial and venous systems causing high flow through AVM and high-pressure arterialisation of draining veins
No cause found (15%)
Rare
- Bleeding disorder
- Mycotic aneurysms - endocarditis
- Acute bacterial meningitis
- Tumours
How well did you know this?
1
Not at all
2
3
4
5
Perfectly
72
Q

What can increase your risk of SAH?

A
Hypertension
Known aneurysm
FHx
Disease that predisposes you to aneurysm
- PKD
- Ehlers Danlos syndrome (hyper-mobile joints with increased skin elasticity)
- Coarctation of aorta
Smoking, bleeding disorders, post-menopausal decreased oestrogen
How well did you know this?
1
Not at all
2
3
4
5
Perfectly
73
Q

What is the pathophysiology of SAH?

A

Most common cause ruptured aneurysm which leads to tissue ischaemia (less blood can reach tissue) as well as rapid raised ICP as blood, acts like space-occupying lesion, puts pressure on brain, resulting in deficits if not resolved quickly

How well did you know this?
1
Not at all
2
3
4
5
Perfectly
74
Q

How do SAH present?

A

Sudden onset severe occipital headache - thunder clap
Depressed level of consciouness
Coma/drowsiness may last for days
Neck stiffness
Kernig’s sign (unable to extend patient’s leg at knee when thigh flexed)
Brudzinski’s sign
Retinal and vitreous bleeds
Papilloedema
Vision loss or diplopia
Focal neurology at presentation (fixed dilated pupils - signs of CN3 palsy)
Marked increase in BP as a reflex to following haemorrhage
Sentinel headache
- May earlier have experienced a sentinel headache due to small warning leak
- Ask about Hx

How well did you know this?
1
Not at all
2
3
4
5
Perfectly
75
Q

What is a differential diagnosis of SAH?

A
Must be differentiated from migraine
50-60% no cause of headache
Meningitis 
Intracerebral bleeds
Cortical vein thrombosis
How well did you know this?
1
Not at all
2
3
4
5
Perfectly
76
Q

How is SAH diagnosed?

A

ABG - to exclude hypoxia
Head CT - gold standard
- Detects > 90% of SAH within 1st 48 hours
- Seen as star shaped lesion due to blood filling in gyro patterns around brain and ventricles
CT angiography if aneurysm confirmed to see extent
Lumbar puncture
- If CT normal but SAH still suspected
- CSF uniformly bloody early on and becomes yellow after several hours due to Hb breakdown (xanthochromia)
- Xanthochromia confirms SAH

How well did you know this?
1
Not at all
2
3
4
5
Perfectly
77
Q

How is SAH treated?

A

Refer all proved SAH to neurosurgeon immediatrly
Maintaun cerebral perfusion keeping well hydrated (IV fluids) and aim for BP < 160mmHg
CCBs to reduce vasospasm and consequent morbidity from ischaemia IV/oral nimodipine
Endovascular coiling
- Preferred to surgical clipping since lower complication rate where possible
- Promotes thrombosis and ablation of aneurysm
- First line treatment where angiography shows aneurysm
Surgery - intracranial stents and balloon remodelling for wide-necked aneurysms

How well did you know this?
1
Not at all
2
3
4
5
Perfectly
78
Q

What are the potential complications of SAH?

A

Rebleeding
Cerebral ischaemia due to vasospasm - can result in permanent deficit
Hydrocephalus due to blockage of arachnoid granulations - requires ventricular or lumber drain
Hyponatraemia

How well did you know this?
1
Not at all
2
3
4
5
Perfectly
79
Q

What is a subdural haemorrhage?

A

Caused by accumulation of blood in subdural space - between arachnoid and dura mater following rupture of bridging vein between cortex and venous sinus (vulnerable to deceleration injury)
Considered very treatable in all those whose conscious level fluctuates and also in those having evolving stroke - especially if on anticoagulants

How well did you know this?
1
Not at all
2
3
4
5
Perfectly
80
Q

How common is subdural haemorrhage?

A

Most common where patient has small brain eg alcoholic or dementia or babies that have suffered trauma, or elderly with brain atrophy making bridging veins more vulnerable
Majority from trauma but trauma often forgotten as so minor or so long ago
Chronic, apparently spontaneous SDH common in elderly and also occurs with anticoagulants

How well did you know this?
1
Not at all
2
3
4
5
Perfectly
81
Q

What can cause subdural haemorrhage?

A

Trauma either due to deceleration due to violent injury or due to dural metastases resulting in bleeding from bridging veins between cortex and venous sinus
Bridging veins bleed and form haematoma between dura and arachnoid reducing pressure and bleeding stops
Days/weeks later haematoma starts to autolyse due to massive increase in oncotic and osmotic pressure thus water sucked into haematoma resulting in haematoma enlargement
Results in gradual rise in intra-cranial pressure over many weeks
Shifting midline strucutres away from side of clot and if untreated leads to eventual tectorial herniation and coning

How well did you know this?
1
Not at all
2
3
4
5
Perfectly
82
Q

What can increase your risk of subdural haemorrhages?

A

Traumatic head injury
Cerebral atrophy/increasing age
Alcoholism, anticoagulation and physical abuse of infant

How well did you know this?
1
Not at all
2
3
4
5
Perfectly
83
Q

How do subdural haemorrhages present?

A

Interval between injury and symptoms can be days to weeks or months
Fluctuating level of consciousness +/- insidious physical or intellectual slowing
Sleepiness
Headache
Personality change
Unsteadiness
Signs of ICP eg headache, vomiting, nausea, seizure and raised BP
Focal neurology eg hemiparesis or sensory loss (occurs later)
Seizures
Stupor, coma, coning
In elderly symptoms will develop slowly as brain has more compliance to raised ICP

How well did you know this?
1
Not at all
2
3
4
5
Perfectly
84
Q

What is a possible differential diagnosis of subdural haemorrhage?

A

Stroke, dementia, CNS masses eg tumours/abscesses
SAH
Extradural haemorrhage

How well did you know this?
1
Not at all
2
3
4
5
Perfectly
85
Q

How are subdural haemorrhages diagnosed?

A

CT head
- Diffuse spreading, hyperdense crescent shaped hemisphere
- As clot ages and protein degradation occurs becomes isodense and eventually hypodense
- Shifting of midline structures
MRI head for subacute and smaller haematomas

How well did you know this?
1
Not at all
2
3
4
5
Perfectly
86
Q

How are subdural haemorrhages treated?

A

Assess and manage ABCs, prioritise head CT
Stabilise patient
Refer to neurosurgeons - irrigation/evacuation via burr twist drill and burr hole craniotomy
Address cause of trauma
IV mannitol to reduce ICP

How well did you know this?
1
Not at all
2
3
4
5
Perfectly
87
Q

What is an extradural haemorrhage?

A

Suspect this after head injury and conscious level falls or is slow to improve or there is a lucid interval
Collection of blood between dura mater and bone usually caused by head injury

How well did you know this?
1
Not at all
2
3
4
5
Perfectly
88
Q

What can cause extradural haemorrhages?

A

Most commonly due to traumatic head injury resulting in fracture of temporal or parietal bone causing laceration of middle meningeal artery - typically after trauma to temple

How well did you know this?
1
Not at all
2
3
4
5
Perfectly
89
Q

What can increase your risk of extradural haemorrhage?

A

Usually occurs in young adults (rare < 2 and > 60)

How well did you know this?
1
Not at all
2
3
4
5
Perfectly
90
Q

How do extra-dural haemorrhages present?

A

Characteristic history - head injury, brief post-traumatic loss of consciousness or initial drowsiness, lucid interval
Severe headache, nausea, vomiting, confusion, seizures due to rising ICP +/- hemiparesis with brisk reflexes
Rapid rise in ICP as epidural space full of blood
Ipsilateral pupil dilates, coma deepens, bilateral limb weakness develops and breathing becomes deep and irregular (brainstem compression)
Decreased GCS and coning
Death due to respiratory arrest
Bradycardia and raised BP late signs

How well did you know this?
1
Not at all
2
3
4
5
Perfectly
91
Q

What is a possible differential diagnosis of extradural haemorrhages?

A

Epilepsy, carotid dissection, CO poisoning
Subdural haematoma
SAH
Meningitis

How well did you know this?
1
Not at all
2
3
4
5
Perfectly
92
Q

How are extradural haemorrhages diagnosed?

A

CT head - gold standard
- Shows hyperdense haematoma that is biconvex and adjacent to skull
- Blood forms more rounded shape compared with sickle shaped subdural haematoma
Skull x-ray - may be normal or show fracture lines crossing course of middle meningeal artery

How well did you know this?
1
Not at all
2
3
4
5
Perfectly
93
Q

How are extra-dural haemorrhages treated?

A

ABCDE emergency management - assess and stabilise patient
Give IV mannitol if increased ICP
Refer to neurosurgery - clot evacuation +/- litigation of bleeding vessel
Maintain airway via intubation and ventilation in unconscious patient

How well did you know this?
1
Not at all
2
3
4
5
Perfectly
94
Q

What is encephalitis?

A

Infection and inflammation of brain parenchyma

How well did you know this?
1
Not at all
2
3
4
5
Perfectly
95
Q

How common is encephalitis?

A

Most frequent in children and elderly - mainly viral caue

More common in immunocompromised

How well did you know this?
1
Not at all
2
3
4
5
Perfectly
96
Q

What can cause encephalitits?

A
Mainly viral 
- Herpes simplex viruses 1 and 2
- Varicella zoster, EBV, cytomegalovirus, HIV, mumps, measles
Non-viral
- Bacterial meningitis
- TB
- Malaria
How well did you know this?
1
Not at all
2
3
4
5
Perfectly
97
Q

What can increase your risk of getting encephalitis?

A

Extremes of age

Immunocompromised

How well did you know this?
1
Not at all
2
3
4
5
Perfectly
98
Q

What is the pathophysiology of encephalitis?

A

Disease that mostly affects frontal and temporal lobes resulting in decreased consciousness, confusion and focal signs

How well did you know this?
1
Not at all
2
3
4
5
Perfectly
99
Q

How does encephalitis present?

A

Whole brain affected - problems with consciousness
Insidious onset (days) or can be abrupt
Triad - fever, headache, altered mental state
Begins with features of viral infection - fever, headaches, myalgia, fatigue, nausea
Progresses to - personality and behavioural changes, decreased consciousness, drowsiness, confusion
Focal neurological deficit - hemiparesis, dysphasia, seizures, raised ICP, midline shift, coma
Signs of meningitis
Hx of travel and/or animal bite

How well did you know this?
1
Not at all
2
3
4
5
Perfectly
100
Q

What is a possible differential diagnosis of encephalitis?

A

Meningitis
Stroke
Brain tumour

How well did you know this?
1
Not at all
2
3
4
5
Perfectly
101
Q

How is encephalitis diagnosed?

A

MRI
- Areas of inflammation and swelling, generally in temporal lobes in HSV encephalitis
- May be midline shifting due to raised ICP
Lumbar puncture
- CSF shows elevated lymphocyte count
- Viral detection by CSF PCR highly sensitive for herpes simplex and varicella zoster
- False negative may occur within first 48 hours symptoms onset
Blood and CSF serology usually helpful

How well did you know this?
1
Not at all
2
3
4
5
Perfectly
102
Q

How is encephalitis treated?

A

If viral - immediate treatment with anti-viral IV acyclovir
Early treatment can reduce mortality and long neuro damage
Anti-seizure medication
If meningitis suspected then emergency IM benzylpenicillin

How well did you know this?
1
Not at all
2
3
4
5
Perfectly
103
Q

What is shingles?

A

Caused by reactivation of varicella zoster virus usually within dorsal root ganglia
Primary infection with VZV causes chickenpox, following which the virus remains latent in sensory ganglia
Development of shingles may indicate decline in cell-mediated immunity such as that due to age or malignancy

How well did you know this?
1
Not at all
2
3
4
5
Perfectly
104
Q

How common is the Varicella zoster virus?

A

90% children have been exposed to chicken pox before 16
Can affect all ages but seen as disease of elderly
Incidence and severity can increase with age
Shingles cannot be caught from contact with person or chickenpox

How well did you know this?
1
Not at all
2
3
4
5
Perfectly
105
Q

What can increase your risk of getting shingles?

A
Increasing age
Immunocompromised
HIV
Hodgkin's lymphoma
Bone marrow transplants
How well did you know this?
1
Not at all
2
3
4
5
Perfectly
106
Q

How does shingles occur?

A

Viral infection affecting peripheral nerves
When latent virus reactivated in dorsal root ganglia travels down affected nerve via sensory root in dermatomal distribution over period 3-4 days
Results in perineural and intramural inflammation
In immunocompromised, most frequent site is throacic nerves followed by ophthalmic division of trigeminal nerve
Can also affect cervical, lumbar and sacral nerve roots
Person with weeping shingles rash can cause chickenpox in non-immune person after close contact

How well did you know this?
1
Not at all
2
3
4
5
Perfectly
107
Q

How does shingles present?

A

Pain and paraesthesia in dermatomal distribution
Rash for days
Malaise, myalgia, headache, fever
Rash - consists of papules and vesicles restricted to same dermatome
- Neuritic pain
- Crust formation and drying occurs over next week with resolution in 2-3 weeks
- Patients infectious until lesions are dried
- Rash doesn’t extend outside dermatome

How well did you know this?
1
Not at all
2
3
4
5
Perfectly
108
Q

What is a possible differential for shingles?

A

Before rash appears pain may come from chest or abdominal pain - cholecystitis or renal stones
Cluster headaches/migraines
Atopic eczema, contact dermatitis or herpes simplex or impetigo

How well did you know this?
1
Not at all
2
3
4
5
Perfectly
109
Q

How is shingles diagnosed?

A

Clinical diagnosis

Rash diagnostic

How well did you know this?
1
Not at all
2
3
4
5
Perfectly
110
Q

How is shingles treated?

A

Oral antiviral therapy begun within 72 hours of rash onset
- Oral acylovir x5 daily
- Or oral valicilovir x2 daily
- Or oral famciclovir x2 daily
Rapid treatment to minimise risk of peripheral herpetic neuralgia
Topical antibiotics treatment for secondary bacterial infection
Analgesia eg ibuprofen for pain

How well did you know this?
1
Not at all
2
3
4
5
Perfectly
111
Q

What are the complications of shingles?

A
Ophthalmic branch of trigeminal nerve - damage will affect sight
Post herpetic neuralgia
- Pain lasting for more than 4 months after developing shingles
- Occurs in 10% patients often elderly
- Burning, intractable pain
- Responds poorly to analgesics
Treat with
- Tricyclic antidepressants
- Anti-epileptics
- Anti-convulsants
How well did you know this?
1
Not at all
2
3
4
5
Perfectly
112
Q

What is meningits?

A

Inflammation of meninges

How well did you know this?
1
Not at all
2
3
4
5
Perfectly
113
Q

How common is meningitis?

A

Occurs in people of all age groups but more common in infants, young children and elderly
Meningococcal disease notifiable to PHE

How well did you know this?
1
Not at all
2
3
4
5
Perfectly
114
Q

What can cause meningitis in adults and children?

A

Neisseria meningitides - gram negative diplococci transmitted by droplet spread
Streptococcus pneumoniae/pneumoncoccus
Less common H influenzae

How well did you know this?
1
Not at all
2
3
4
5
Perfectly
115
Q

What can cause meningitis in pregnant women and older adults?

A

Listeria monocytogenes - found in cheese

How well did you know this?
1
Not at all
2
3
4
5
Perfectly
116
Q

What can cause meningitis in neonates?

A

E coli

Group B haemolytic stretpcoccus - strep agalactiae

How well did you know this?
1
Not at all
2
3
4
5
Perfectly
117
Q

What can cause meningitis in immunocompomised?

A
Cytomegalovirus
Cryptococcus neoformans (fungi)
TB 
HIV
Herpes simplex virus
How well did you know this?
1
Not at all
2
3
4
5
Perfectly
118
Q

What can increase your risk of meningitis?

A
Intrathecal drug administration
Immunocompromised
Elderly
Pregnant
Bacterial endocarditis
Crowding - military base, university students
Diabetes
Malignancy
IVDU
How well did you know this?
1
Not at all
2
3
4
5
Perfectly
119
Q

What is the pathophysiology of meningitis?

A

Microorganisms reach meninges either by direct extension from ears, nasopharynx, cranial injury or congenital meningeal defect or bloodstream
Immunocompromised patients at risk of infection with unusual organisms
Acute bacterial meningitis
- Typically sudden
- Can lead to meningococcal septicaemia when bacteria invade blood
- Presence of endotoxin leads to inflammatory cascade
- Petechial rash and signs of sepsis
- Pia arachnoid congested with polymorphs
- Layer of pus which may organise to form adhesions causing cranial nerve palsies and hydrocephalus
Chronic infection eg TB
- Brain covered in viscous grey-green exudate with numerous meningeal tubercules
Viral
- Predominantly lymphcytic inflammatory CSF reaction without pus formation
- Little or no cerebral oedema unless encephalitis develops

How well did you know this?
1
Not at all
2
3
4
5
Perfectly
120
Q

How does meningitis present?

A

Triad - headache, neck stiffness, fever
In acute bacterial
- Onset sudden
- Papilloedema bilateral occuring over hours-weeks
- Intense malaise, fever, rigors, severe headache, photophobia, vomiting
- Irritable and prefers to lie still
- Neck stiffness, positive Kernig’s and Brundzinski’s sign
- Meningococcal septicaemia - non-blanching petechial and purpuric rash
- Altered mental state with high fever
- Seizures and focal CNS signs
- Progressive drowsiness, materialising signs, cranial nerve lesions indicate complication
Viral meningitis
- Benign, self-limiting condition lasting 4-10 days
- Headache may follow for some months
Chronic
- TB
- Long Hx and vague symptoms of headache, anorexia and vomiting
- Sings of meningeal tried often absent/late

How well did you know this?
1
Not at all
2
3
4
5
Perfectly
121
Q

What is a possible differential diagnosis of meningitis?

A

Aseptic meningitis due to tumour
SAH
Encephalitis

How well did you know this?
1
Not at all
2
3
4
5
Perfectly
122
Q

How is meningitis diagnosed?

A

Blood cultures before lumbar puncture
Blood tests - FBC, U&E, CRP, serum glucose
Lumbar puncture - if unable to perform within 30 mins give empirical antibiotics
- Send for microscopy and sensitivity
- Can give headache, paraesthesia, CSF leak and damage to spinal cord
CT head
- To exclude lesions
Throat swabs - bacterial and viral
Pneumococal and meningococcal serum PCR

How well did you know this?
1
Not at all
2
3
4
5
Perfectly
123
Q

How do you differentiate between bacterial, TB and viral meningitis on lumbar puncture results?

A
Bacteria
- Neutrophil polymorphs
- High protein
- Low glucose
TB
- Lymphocytes
- High protein
- Low/normal glucose
Viral
- Lymphocytes
- Normal protein
- Normal glucose
How well did you know this?
1
Not at all
2
3
4
5
Perfectly
124
Q

How do you treat meningococcal septicaemia?

A

IV benzylpenicillin or cefotaxime immediately
Lumbar puncture not performed due to risk of coning and raised ICP
Diagnosis confirmed by blood cultures

How well did you know this?
1
Not at all
2
3
4
5
Perfectly
125
Q

When would you do a head CT before a lumbar puncture?

A
Age > 60
Immunocompromised
Hx of CNS disease
New onset/recent seizures
Decreasing conscious levels GCS < 14
Focal neurological signs
Papilloedema
How well did you know this?
1
Not at all
2
3
4
5
Perfectly
126
Q

How do you treat bacterial meningitis?

A

Start antibiotics before tests come back if suspected
IV cefotaxime or ceftriaxone
If over 50/immunocompromised add IV amoxicillin to cover listeria
Consider steroids to reduce cerebral oedema
IV vancomycin if return traveller
Prophylaxis for contacts
- Oral ciprofloxacin stat
- Oral rifampicin - cannot give when pregnant

How well did you know this?
1
Not at all
2
3
4
5
Perfectly
127
Q

What are the potential complications of meningitis?

A

Hearing loss
Seizures
Developmental problems

How well did you know this?
1
Not at all
2
3
4
5
Perfectly
128
Q

What does paresis mean?

A

Impaired ability to move a body part in response to will

Weakness

How well did you know this?
1
Not at all
2
3
4
5
Perfectly
129
Q

What does paralysis mean?

A

Ability to move a body part in response to will completely lost

How well did you know this?
1
Not at all
2
3
4
5
Perfectly
130
Q

What does ataxia mean?

A

Incoordination

Willed movements clumsy, ill-directional or uncontrolled

How well did you know this?
1
Not at all
2
3
4
5
Perfectly
131
Q

What are involuntary movements?

A

Spontaneous movement of body part, indefinitely of will

How well did you know this?
1
Not at all
2
3
4
5
Perfectly
132
Q

What is apraxia?

A

Disorder of consciously organised pattern of movement or impaired ability to recall acquired motor skills

How well did you know this?
1
Not at all
2
3
4
5
Perfectly
133
Q

What is a lower motor neurone?

A

Located in anterior horns of spinal cord and in CN nuclei in brainstem

How well did you know this?
1
Not at all
2
3
4
5
Perfectly
134
Q

What are the potential sites of damage along a motor pathway?

A
Motor nuclei of cranial nerves
Motor neurones in spinal cord
Spinal ventral roots
Peripheral nerves
Neuromuscular junction
Muscle
How well did you know this?
1
Not at all
2
3
4
5
Perfectly
135
Q

What are the UMN damage signs?

A

Everything goes up
Spasticity (increased muscle tone)
Brisk reflexes
Plantars upturned on stimulation - positive Babinski’s sign
Characteristic pattern of limb muscle weakness (pyramidal pattern)
- Upper limbs extensor muscles weaker than flexors
- Lower limb flexors weaker than extensors
- Finer more skillful movement impaired

How well did you know this?
1
Not at all
2
3
4
5
Perfectly
136
Q

What are the LMN damage signs?

A
Everything goes down
Muscle tone reduced - flaccid
Muscle wasting
Fasciculation - visible spontaneous contraction of motor units - not enough to diagnose LMN, need weakness too
Reflexes depressed/absent
How well did you know this?
1
Not at all
2
3
4
5
Perfectly
137
Q

What is motor neurone disease?

A

Cluster or major degenerative diseases charaterised by selective loss of neurones in motor cortex, cranial nerve nuclei and anterior horn cells

How well did you know this?
1
Not at all
2
3
4
5
Perfectly
138
Q

How common is MND?

A

Relatively uncommon
More common in men
Median age onset 60
Often fatal in 2-4 years

How well did you know this?
1
Not at all
2
3
4
5
Perfectly
139
Q

What can cause MND?

A
Usually sporadic and of unknown cause
No established factors
5-10% familial
- Linked to mutation in free radical scavenging enzyme superoxide dismutase
- Also linked to TDP-42, C9LRF72 and FUS
How well did you know this?
1
Not at all
2
3
4
5
Perfectly
140
Q

What occurs in MND?

A

Degenerative condition affecting motor neurones, namely anterior horn cells
Relentless and unexplained destruction of UMN and anterior horn cells in brain and spinal cord
Causes both UMN and LMN dysfunction
No sensory loss of sphincter disturbance
Never affects eye movements
Causes reactive oxygen species which damage DNA, lipids and proteins
Most patients die within 3 years from respiratory failure as a result of bulbar palsy and pneumonia

How well did you know this?
1
Not at all
2
3
4
5
Perfectly
141
Q

How does MND present?

What are the 4 main clinical patterns?

A

Amyotrophic lateral sclerosis (UMN and LMN)
- Weakness and UMN neurone signs
- Split hand sign (thenar wasting, less hypothenar wasting)
- Cramps
- Wrist and foot drop
Progressive muscular atrophy (LMN)
- Weakness, muscle wasting, fasciculations
- Affects distal before proximal
Progressive bulbar palsy (LMN)
- Lower CN 9-12 and nuclei
- Dysarthria, dysphasia, nasal regurgitation (chocking)
- LMN lesion of tongue and muscles of talking, swallowing, flaccid fasciculating tongue, jaw jerk normal/absent, speech quiet, hoarse of nasal
Primary lateral sclerosis (LMN)
- Loss of beta cells in motor cortex
- UMN signs
- No cognitive decline

How well did you know this?
1
Not at all
2
3
4
5
Perfectly
142
Q

When may you think of MND?

A

> 40, stumbling spastic gait, foot drop +/- proximal myopathy, weak grip and shoulder abduction or aspiration pneumonia
Frontotemporal dementia
Spread of symptoms
Rectal and bladder sphincters and oculomotor muscles spared
No sensory symptoms

How well did you know this?
1
Not at all
2
3
4
5
Perfectly
143
Q

What is a differential diagnosis of MND?

A
MS/polyneuropathies
Myasthenia gravis
Diabetic amyotrophy
GBS
Spinal cord tumour
How well did you know this?
1
Not at all
2
3
4
5
Perfectly
144
Q

How is MND diagnosed?

A

Based on clinical findings
- Definite - LMN and UMN signs in 3 regions
- Probable - LMN and UMN in 2 regions
- Probably with lab support - LMN and UMN signs in 1 region or UMN in more than one region + electomyography shows acute denervation in more than 2 limbs
- Possible - LMN and UMN signs in 1 region
- Suspected - LMN/UMN sign only in 1 or more
Brain/spinal cord MRI (excludes structural causes)
Lumbar puncture - excludes inflammatory causes
Nerve conduction studies + electromyography
Denervation of muscles due to degeneration of LMNs confirmed by EMG

How well did you know this?
1
Not at all
2
3
4
5
Perfectly
145
Q

How is MND treated?

A

Anti-glutametergic drugs
- Oral riluzole (Na+ channel blocker inhibiting glutamate), prolongs life by 3 months
- Raised LFTs so monitor
- S/E vomiting, raised pulse, headache, vertigo
Drooling - oral propantheline or amitriptyline
Dysphasia
- Blend food
- Nasogastric tube
- percutaneous catheter gastrostomy
Spasms - oral baclofen
Non-invasive ventilation if respiratory failure
Analgesia - NSAIDs or opioids
Specialist MDT supprot - neurologist, speech and language therapists, occupational therapist, specialist nurse, physiotherapise

How well did you know this?
1
Not at all
2
3
4
5
Perfectly
146
Q

What is Parkinson’s disease?

A

Degenerative movement disorder caused by reduction in dopamine in substantia nigra
Characterised by triad of rigidity, bradykinesia, resting tremor

How well did you know this?
1
Not at all
2
3
4
5
Perfectly
147
Q

How common is Parkinson’s?

A

Increasing prevalence with age
Peak age of onset 55-65
More common in men

How well did you know this?
1
Not at all
2
3
4
5
Perfectly
148
Q

What can cause Parkinson’s?

A

Idiopathic
Drug induced
Combination of
- Environmental factors - pesticides, methyl-phenyl tetrahydropyridine in illegal opiates
- Parkinson’s genes
- Oxidative stress and mitochondrial dysfunction

How well did you know this?
1
Not at all
2
3
4
5
Perfectly
149
Q

What can increase your risk of getting Parkinson’s?

A

Male
Increasing age
FHx
Being non-smoker have higher risk

150
Q

How does Parkinson’s occur?

A

Substania nigra produces dopamine
Parkinson’s results from mitochondrial dysfunction and oxidative stress
Results from progressive degeneration of dopaminergic neurones from pars compacta of SN in midbrain that projects to striatum of basal ganglia
Reduced striatal dopamine levels due to loss of dopaminergic neurones
Less dopamine so thalamus inhibited resulting in decrease in movement
Neuronal inclusions called Lewy-Bodies present in Parkinson’s and become gradually more widespread as condition progresses, spreading from lower brainstem to midbrain and then cortex
Degeneration of other basal ganglia nuclei
Extent of nigrostriatal dopaminergic cell loss correlates with degree of akinesia

151
Q

How does Parkinson’s present?

A

Onset gradual and often presents with impaired dexterity of unilateral foot drop
Asymmetry
Before motor symptoms - anosmia, depression/anxiety, aches and pains, REM sleep disorders, urinary urgency, hypotension, constipation
Tremor
- Worse at rest, asymmetrical, most obvious in hands, improved with voluntary movements, worsened by anxiety, 4-6 cycles/sec, issue with repetitive hand movements
Rigidity - increased tone in limbs and trunk, limbs resist passive extension throughout movement, over entire radius of joint movement, can cause pain and problems with turning in bed
Bradykinesia/hypokineaia - slow to imitate movement, reduce blink rate, monotonous hypophonic speech, smaller writing
Gait - reduced asymmetrical arm swing, narrow gait, stooped posture and small steps, shuffling, narrow base
Difficulty with fine movements
Drooling and swallowing difficulty late
Depression, constipation, increased urinary frequency

152
Q

What is a possible differential for Parkinsons?

A
Benign essential tremor - worse on movement, rare at rest
Multiple cerebral infarcts
Lewy-body dementia
Drug induced
Wilson's disease
Trauma
All dopamine antagonists
153
Q

How do you treat benign essential tremor?

A

Beta-blockers
Primidone
Gabapentin

154
Q

How is Parkinson’s diagnosed?

A

Clinical
Confirmation by response to L-dopa
MRI head - initially normal but slowly shows atrophy, exclude tumours or normal pressure hydrocephalus
If any of these present at beginning, sign probably not Parkinson’s
- Dementia, incontinence, symmetry, early falls

155
Q

How is Parkinson’s treated?

A

Compensate for loss of dopamine
Explain disease slowly progressive and although incurable, amenable to palliation
Balance problems, speech, gait disturbance don’t respond to medication so physio
Encourage physical activity
Oral levodopa with decarboxylase inhibitor
Doesn’t alter disease progression, offers symptom relief
Only started when absolutely necessary
Dopamine agonists - oral ropinirole or pramiprexole
Monoamine oxidase B inhibitors oral selefiline or rasagline
Catechol-O-methyl transferase inhibitors eg oral entacapone or tolcapone
Neuropsychiatric complications eg depression, dementia, psychosis
Deep brain stimulation to help those who are partly dopamine responsive
Surgical ablation of overactive basal ganglia circuits

156
Q

What are the motor defects of long term L-dopa?

A

Reduced efficacy over time even if dose increase
On-dyskinesias
Off-dyskinesias
Freezing

157
Q

What is multiple sclerosis?

A

Chronic autoimmune, T cell mediated inflammatory disorder of CNS in which there are multiple plaques of demyelination within brain and spinal cord, occurring sporadically over years
Disease of CNS with oligodendroytes targeted, not schwann cells of PNS affects white matter of brain

158
Q

How common is multiple sclerosis?

A
Begins in early adulthood
More common in women
Presentation typically between 20-40
Presentation after 60 rare
More common in white populations
More common further from equator you go - rare in tropical countries
159
Q

What are the different types of MS?

A

Relapsing and remitting
- Most common pattern
- Symptoms occur in attacks with onset over days and typically recovery either partial or complete over weeks
- Periods of good health/remission followed by sudden symptoms or relapses
- Patients may accumulate disability over time if don’t fully recover from relapses
Secondary progressive MS
- Follows on from relapsing and remitting MS
- Late stage of MS that consists of gradually worsening symptoms with fewer remissions
- 75% patients with relapsing-remitting MS will eventually evolve into secondary progressive MS by 35 years after onset
Primary progressive MS
- Gradually worsening disability without relapses or remission
- Typically presents later and associated with fewer inflammatory changes on MRI

160
Q

What causes MS?

A

Not understood
Combination of genetic and environmental factors
- Exposure to EBV in childhood
- Low levels of sunlight and vitamin D

161
Q

What can increase you risk of getting MS?

A

Female
White
Living far from equator - Southern and Northern hemispheres (vit D levels)

162
Q

What is the pathophysiology of MS?

A

Autoimmune mediated demyelination at multiple CNS sites resulting in discrete plaques of demyelination affecting white matter
Thought to be T cell mediated (activate B cells to produce auto-antibodies against myelin)
Once T lymphocytes cross BBB cause destruction in brain
Results in demyelination and thus conduction disruption along axons
Although myelin sheath does regenerate, new myelin less efficient and temperature dependent - when exposed to high heat, conduction through new myelin decreases drastically
Plaques of demyelination perivenular occur everywhere in CNS
PNS not affected as Schwann cells unaffected due to different antigens
Repeated demyelination leads to axonal loss and incomplete recovery between attacks
Poor healing results in relapsing and remitting symptoms
Multiple areas of sclerosis form along neurons slowing conduction of signals and impairing movement and/or sensations

163
Q

In MS, where within CNS is most commonly affected?

A
Optic nerves
Around ventricles of brain
Corpus callosum
Brainstem and cerebellar connections
Cervical cord
164
Q

How does MS present?

A

In young adults 20-40
Monosymptomatic initially
Symptoms may worsen with heat/exercise as new myelin inefficient and doesn’t perform well in heat
Unilateral optic neuritis - pain in one eye on movement, reduced central vision
Numbness or tingling of limbs
Leg weakness
Brainstem demyelination - diplopia, vertigo, facial numbness/weakness, dysarthria/dysphasia
Clumsy/useless hand or limb movements due to loss of proprioception
Cerebellar symptoms such as ataxia
Trigeminal neuralgia
Constipation
Spasticity and weakness
Intention tremor
Bladder dysfunction - urgency, frequency, incontinence
Sexual dysfunction - erectile dysfunction
Cognitive decline
Amnesia

165
Q

What is a possible differential for MS?

A

Hereditary spastic paraplegia
Cerebral variant of SLE
Sarcoidosis
HIV

166
Q

How is MS diagnosed?

A

2/more attacks affecting different parts of CNS
Exclude differentials with FBC, inflammatory markers, U&Es, LFTs, glucose, HIV serology, auto-antibodies, Ca2+, vitamin B12
MRI brain and cord
- Diagnostic if Hx
- 95% periventricular lesions
- 90% discrete white matter abnormalities
- Multiple scattered plaques usually seen
Lumbar puncture
- CSF exam shows oligoclonal IgG bands in over 90% cases
- CSF cell count may be raised
Electrophysiology - delayed nerve conduction suggests demyelination

167
Q

How is MS treated?

A

Encourage stress-free life to help reduce number of lesions
If poor diet and sun exposure give vit D
Acute relapse
- IV methylprednisolone for less than 3 days
Frequent relapse
- SC interferon 1B or 1A anti-inflammatory cytokines
- Disease modifying agents
- Monoclonal antibodies - IV alemtuzumab or IV natalizumab
Symptomatic treatment of spasticity
- Physio
- Baclofen
- Tizanidine
- Botox injections
- Urinary urgency and frequency
- Incontinence
Aggressive treatment - stem cell transplant
Life expectancy for people with MS 5-10 years below average often die from aspiration pneumonia due to dysphagia and swallowing difficulties

168
Q

What is epilepsy?

A

Recurrent tendency to spontaneous, intermittent, abnormal electrical activity in part of brain, manifesting in seizures
Ongoing liability to recurrent epileptic seizures
Paroxysmal/unprovoked event in which changes of behaviour, sensation or cognitive processes are caused by excessive hypersynchronous
neuronal discharged in brain
Chronic disorder - need at least 2 seizures to be defined as epileptic

169
Q

What are convulsions?

A

Motor signs of electrical discharges
Many would have seizures in abnormal metabolic circumstances (hyponatraemia, hypoxia)
But would not normally be said to have epilepsy

170
Q

How common is epilepsy?

A

Common
Incidence age-dependent, highest at extremes of life with most cases starting before 20 or after 60
Can often go into remission
Seizures usually last 30-120 seconds

171
Q

What are the different types of seizures?

A

Primary generalised
- Simultaneous onset of electrical discharge throughout whole cortex (involving both hemispheres), with no localising features referable to only one hemisphere
- Bilateral symmetrical and synchronous motor manifestations
- Always associated with loss of consciousness or awareness
Partial/focal seizures
- Focal onset, with features referable to a part of one hemisphere
- Often seen with underlying structural disease
- Electrical discharge restricted to a limited part of cortex of one cerebral hemisphere
- May later become generalised eg secondary generalised tonic-clonic seizures

172
Q

What can cause epilepsy?

A
2/3 idiopathic often familial
Cortical scarring
- Head injury years before onset
- Cerebrovascular disease
- CNS infection
Space occupying lesion
Tuberous sclerosis
Alzheimer's or dementia
Alcohol withdrawal
173
Q

What can increase your risk of getting epilepsy?

A
FHx
Premature born babies who small for age
Abnormal blood vessels in brain
Alzheimer's or dementia
Use of drugs
Stroke/brain tumour/infection
174
Q

What are the elements of a seizure?

A

Prodrome
- Lasting hours or days may rarely precede seizures
- Not part of seizure, results in change of mood or behaviour
Aura
- Part of seizure where patient aware and may precede other manifestation
- Strange feeling in gut, deja vu or strange smells or flashing lights
- Implies a partial (focal) seizure often not necessarily from temporal lobe
Post-ictally (after)
- Headache, confusion, myalgia, sore tongue
- Temporary weakness after a focal seizure in motor cortex - Todd’s palsy
- Dysphasia following a focal seizure in temporal lobe

175
Q

What are the different types of primary generalised seizures?

A
Generalised tonic-clonic seizure
Typical absence seizure
Myoclonic seizure
Tonic seizure
Atonic seizure
176
Q

What are the types of partial/focal seizures?

A

Simple partial seizure
Complex partial seizure
Partial seizure with secondary generalisation
(Involves different lobes)

177
Q

What are the symptoms of a generalised tonic-clonic seizure?

A

Often no aura
Loss of consciousness
Tonic phase - rigid, stiff limbs, falls to floor if standing
Clonic phase - generalised, bilateral, rhythmic muscle jerking lasting secs-mins
Eyes remain open, tongue often bitten
Incontinence of urine/faeces
Followed by period of drowsiness, confusion, coma for several hours

178
Q

What are the symptoms of a typical absence seizure?

A

Usually disorder of childhood
Child ceases activity, stares and pales for a few seconds
Suddenly stops talking mid-sentence and then carries on where left off
Often don’t realise they had an attack
On EEG - 3Hz spike and wave activity
Tend to develop generalised tonic-clonic seizures later in life

179
Q

What are the symptoms of a myoclonic seizure?

A

Sudden isolated limb jerk, face or trunk

Patient suddenly thrown to ground, or have violently disobedient limb

180
Q

What are the symptoms of a tonic seizure?

A

Sudden sustained increased tone with characteristic cry/grunt
Intense stiffening of body (tonic)
Stiffening not followed by jerking

181
Q

What are the symptoms of an atonic seizure?

A

Sudden loss of muscle tone and cessation of movement resulting in fall

182
Q

What are the symptoms of a simple partial seizure?

A

Not affecting consciousness or memory
Awareness unimpaired with focal motor, sensory, autonomic or psychic symptoms
No post-ictal symptoms

183
Q

What are the symptoms of a complex partial seizure?

A

Affecting awareness of memory before, during or immediately after seizure
Most commonly arise from temporal lobe
Post-ictal confusion common with seizures arising from temporal lobe whereas recovery rapid after seizure in frontal lobe

184
Q

What are the symptoms of a partial seizure with secondary generalisation?

A

Electrical disturbance starts focally (as either simple or complex partial seizure) then spreads widely causing a secondary generalised seizure which is convulsive

185
Q

What are the characteristics of temporal lobe seizures?

A

Aura
Anxiety or out of body experience
Automatisms eg lip smacking, chewing, fiddling

186
Q

What are the characteristics of frontal lobe seizures?

A

Motor features such as posturing or peddling movements of leg
Jacksonian March - seizure moves up and down motor homunculus starting at face/thumb
Post-ictal Todd’s palsy

187
Q

What are the characteristics of parietal lobe seizures/

A

Sensory disturbances - tingling/numbness

188
Q

What are the characteristics of occipital lobe seizures?

A

Visual phenomena eg spots, lines or flashes

189
Q

What signs suggest epilepsy rather than syncope?

A
Tongue biting
Head turning
Muscle pain
Loss of consciousness
Cyanosis
Post-ictal symptoms
190
Q

What signs suggest syncope rather than epilepsy?

A

Prolonged upright position
Sweat prior to loss of consciousness
Nausea

191
Q

How do you differentiate between a non-epileptic seizure and an epileptic seizure?

A

Non-epileptic seizures are situational
Longer, closed mouth/eyes during tonic-clonic movements, pelvic thrusting
Do not result from sleep, no incontinence or tongue biting
Pre-ictal anxiety symptoms in non-epileptic seizure

192
Q

What is a possible differential for epilepsy?

A
Postural syncope
Cardiac arrhythmia
TIA
Migraine
Hyperventilation
Hypoglycaemia
Panic attacks
Non-epileptic seziure
193
Q

How is epilepsy diagnosed?

A

At least 2 or more unprovoked seizures occurring > 24 hours apart
EEG - not diagnostic
- Performed to support diagnosis
- Helps to determine type of seizure and what epilepsy syndrome
- Frequently normal between attacks and false-positive may be detected in non-epileptics
MRI
- With imaging of hippocampus
CT head
- Used in emergency to look for space occupying lesion
- Used to identify or exclude abnormalities that could be causing symptoms
Bloods
- FBC, electrolytes, Ca2+, renal function, liver function, urine biochemistry, blood glucose levels
- Rules out metabolic causes
Genetic testing

194
Q

How is epilepsy treated?

A

Emergency measures
- ABCDE
- Check glucose
- Prolonged seizure or repeated seizures treated with rectal/IV diazepam or lorazepam
- IV phenytoin loading
- If still fitting then anaesthetist involvement for anaesthetic and ventilation
Generally drugs not advised after one fit unless risk of recurrence high
Resistant to drug treatment in 1/3 patients
Possible drug treatments
- Oral sodium valproate
- Oral lamotrigine
- Oral carbamazepine

195
Q

What is spinal cord compression?

A

Compression of spinal cord resulting in UMN signs and specific symptoms depending on where compression is

196
Q

What can cause spinal cord compression?

A

Vertebral body neoplasms - most common
- Secondary malignancy commonly from lung, breast, prostate, myeloma and lymphoma
Spinal pathology
- Disc herniation - nucleus pulposus moved out through annulus resulting in pressure on nerve root and pian
- Disc prolapse - nucleus pulosus presses against annulus
Rarer - infections, haematoma, primary spinal cord tumour

197
Q

How does spinal cord compression present?

A

Spinal or root pain may precede leg weakness and sensory loss
Progressive weakness of legs with UMN signs
Onset may be acute or chronic depending on cause
Arm weakness often less severe
Bladder sphincter involvement late and manifests as hesitancy, frequency, and later as painless retention
Sensory loss below level of lesion
Look for motor, reflex and sensory level with normal findings above level of lesion
LMN signs at level and UMN below level
Depends on level of lesion
L5/S1 sciatica

198
Q

What is a possible differential diagnosis for spinal cord compression?

A
Transverse myelitis
Multiple sclerosis
Cord vasculitis
Trauma
Dissecting aneurysm
199
Q

How do you diagnose spinal cord compression?

A

Imaging (don’t delay) - irreversible paraplegia may follow if cord not decompressed
Speed of imaging should be parallel to rate of clinical progression
MRI - gold standard - identifies cause and site
Biopsy/surgical exploration to identify masses
Blood tests - FBC, ESR, B12, U&Es, syphilis serology, LFT, PSA
CXR - if TB or malignancy

200
Q

How do you treat spinal cord compression?

A

If malignancy - IV dexamethasone to reduce inflammation and oedema around malignancy and consider chemo/radiotherapy
Epidural abscess - surgically decompressed and antibiotics
Refer to neurosurgeons
- Epidural steroid injection for leg pain
- Surgical decompression of cord
- Laminectomy - removal of lamina/spongy tissue between discs to relieve pressure
- Microdiscectomy - removal of herniated tissue from disc

201
Q

What is cauda equina syndrome?

A

Medical emergency
Cauda equina formed by nerve roots caudal to level of termination of spinal cord at L1/2
Spinal damage caudal to L1
Flaccid and areflexic weakness

202
Q

How common is cauda equina syndrome?

A

Rare, occurring mainly in adults but can occur at any age

Common cause - lumbar disc herniation at L4/5 and L5/S1

203
Q

What can cause cauda equnia syndrome?

A
Herniation of lumbar disc 
Tumours/metastases
Trauma
Infection
Spondylolisthesis
Post-op haematoma
204
Q

How does cauda equnia syndrome present?

A

Major different between cauda equina and lesions higher up - leg weakness is flaccid and areflexic (LMN)
Sciatic pain, numbness, tingling sensation that radiates from lower back and travels down one of legs to foot and toes
Bilateral sciatica
Saddle anaesthesia
Bladder/bowel dysfunction
Erectile dysfunction

205
Q

What is a possible differential of cauda equina syndrome?

A

Conus medullaris syndrome
Vertebral fracture
Peripheral neuropathy
Mechanical back pain

206
Q

How is cauda equina syndrome diagnosed?

A
MRI to localise lesion
Knee flexion to test L5-S1
Ankle plantar flexion - test S1-2
Straight leg raising - L5 -S1 root problem
Femoral stretch test - L4 root problem
207
Q

How is cauda equina syndrome treated?

A

Refer to neurosurgeon ASAP to relieve pressure or risk irreversible paralysis/sensory loss/incontinence

  • Microdisectomy
  • Epidural steroid injection
  • Surgical spine fixation
  • Spinal fusion
208
Q

What is a polyneuropathy?

A

Disorders of peripheral or cranial nerves, whose distribution is usually symmetrical and widespread
Often with distal weakness and sensory loss - glove and stocking

209
Q

How are polyneuropathies classified?

A
By course - acute/chronic
By function - sensory/motor/autonomic/mixed
By pathology - demyelination/axonal degeneration/both
Mostly motor
- GBS
- Lead poisoning
- Charcot-Marie-Tooth syndrome
Mostly sensory
- DM
- Renal failure
- Leprosy
210
Q

What can cause polyneuropathies?

A

Metabolic - DM, renal failure. hypothyroidism, hypoglycaemia
Vaculitides - polyarteritis nodsa, rheumatoid arthritis, Wegner’s granulamatosis
Malignancy - paraneoplastic syndrome, polycythaemia rubra vera
Inflammatory - GBS, sarcoidosis
Infections - leprosy, HIV, syphilis, Lyme disease
Nutritional - decreased Vit B12, B1, E, B6, folate
Inherited syndromes - Charcot-Marie-Tooth, porphyria
Drugs/toxins - lead, arsenic, alcohol, vincristine, cisplatin, metronidazole

211
Q

How do sensory polyneuropathies present?

A

Numbness, paraesthesia
Affects extremities first - glove and stocking
Difficulty handling small objects such as buttons
Signs of trauma eg burns or joint deformation
Diabetic and alcoholic neuropathies typicalyl painful

212
Q

How do motor polyneuropathies present?

A

Often progressive - may be rapid
Weak or clumsy hands
Difficulty walking - falls and stumbling
Difficulty breathing - reduced vital capacity
LMN lesion - wasting and weakness most marked in distal muscle of hands and feet
Reflexes are reduced/absent

213
Q

How are polyneuropathies diagnosed?

A

Hx vital - be clear about time course, precise nature of symptoms and any preceding or associated events
- Diarrhoea and vomiting before GBS
- Weight loss in cancer
- Arthralgia from connective tissue disease
Ask about travel, alcohol and drug use, sexual infections and FHx
If palpable nerve thickening - leprosy or Charcot-Marie-Tooth
Examine other systems for clues to causes
FBC, ESR, glucose, U&Es, LFT, TSH, B12
ANA, ANCA, anti-CCP
CXR
Urinalysis
Lumbar puncture and specific genetic testing for inherited neuropathies

214
Q

How are polyneuropatheis treated?

A

Treat cause
Foot care and shoe choices important to minimise trauma in sensory
Splinting of joints help prevent contractures in prolonged paralysis
For vasculitis causes - steroids, immunosuppressants
Treat neuropathic pain with oral amitriptyline or oral nortirptyline

215
Q

What is Guillanin-Barre syndrome?

A

Acute inflammatory demyelinating ascending polyneuropathy affecting the peripheral nervous system following an upper respiratory or GI tract infection

216
Q

How common is GBS?

A

More common in men
Peak ages 15-35 years and 50-75 years
Most common acute polyneuropathy

217
Q

What can cause GBS?

A
Campylobacter jejuni
Cytomegalovirus
Mycoplasma
Zoster
HIV
EBV
In some cases, no obvious infection can be found
218
Q

What can increase your risk of GBS?

A

History of respiratory GI infections 1-3 weeks prior to onset
Vaccinations have been implicated
Post-pregnancy - incidence decreases during pregnancy but increase in months after delivery

219
Q

What is the pathophysiology of GBS?

A

Usually triggered by infection
Thought that infectious organisms share same antigens as those Schwann cells such as ganglioside GM and GQ1b leading to autoantibody mediated nerve cell damage formation via molecular mimicry
Nerve cell damage consists of damage to Schwann cells and thus demyelination resulting in reduction in peripheral nerve condition resulting in an acute polyneuropathy

220
Q

How does GBS present?

A

1-3 weeks post infection as symmetrical ascending muscle weakness starts - may advance quickly, affecting all limbs at once and can lead to paralysis
Proximal muscles more affected eg trunk, respiratory, cranial nerves
In 20% respiratory muscles and facial muscles are affected - respiratory involvement required ITU admission
Pain common eg back and limb
Sensory signs include paraesthesia by very few sensory signs
Reflexes lost early in illness
Autonomic features such as sweating, raised pulse, BP changes, arrhythmias may be present
Progressive phase of up to 4 weeks, followed by recovery

221
Q

What is a possible differential diagnosis of GBS?

A

Other causes of acute paralysis

  • Hypokalaemia
  • Stroke
  • Brainstem compression
  • Encephalitis
  • Spinal cord compression
  • Poliomyelitis
  • Vasculitis
  • Myasthenia gravis
222
Q

How is GBS diagnosed?

A

Nerve conduction studies
- Diagnostic if matches with clinical exam
- Shows signs of conduction, prolonged distal motor latency +/- conduction block
Lumbar puncture at L4
- CSF has raised protein but normal WCC
Spirometry
- To monitor FVC if there is respiratory involvement
- Decreased FVC indicated need to admit to ITU to maintain airways

223
Q

How is GBS treated?

A

If FVC < 1.5L/80% then ventilated and admit to ITU - monitor FVC 4 hourly
IV immunoglobulins for 5 days
- Decreases duration and severity of paralysis
- Contraindicated in patients with IgA deficiency (can cause allergic reaction)
Plasma exchange
LMW heparin and compression stockings to reduce risk of venous thrombosis
Prognosis good with 85% making complete/near complete recovery
105 unable to walk alone at 1 yr
Mortality 10%

224
Q

What is myasthenia gravis?

A

Autoimmune disease against nicotinic acetylcholine receptors in neuromuscular junction

225
Q

How common is myasthenia gravis?

A

More common in women - although over 50 more common in men
Peak age of incidence at 30 in women
Peak age of incidence at 60 in men

226
Q

What can cause myasthenia gravis?

A

If under 50 and MG commoner in women - associated with other autoimmune disease
If over 50 and MG commoner in men, associated with thymic atrophy or thymic tumour, RA and SLE
Transient MG sometimes caused by D-penicillamine treatment for Wilson’s disease

227
Q

What is the pathophysiology of MG?

A

Autoimmune disease mediated by antibodies to nicotinic acetylcholine receptor - anti-AChR antibodies interfering with neuromuscular junction via depletion of working post-synaptic receptor sites
Achieved by immune complex deposition of anti-AChR IgG and complement at post-synaptic membranes, causing interference and destruction of receptors
Both B and T cells are implicated
Blocks the excitatory effect of ACh on nicotinic receptors resulting in muscle weakness - results in less nicotinic receptors

228
Q

How does myasthenia gravis present?

A

Increasing muscular fatigue
Muscle groups affected in order
- Extra-ocular
- Bulbar
- Face
- Neck
- Trunk
Limb muscles, speech and facial expression commonly affected
Look for ptosis, diplopia and myasthenic snarl on smiling
Respiratory difficulties can occur in generalised myasthenia
Tendon reflexes are normal but may be fatiguable
On examination to elicit fatiguability
- Ask patient to count to 50 (becomes less audible)
- Hold finger up high and ask patient to keep looking at it without lifting head up - after a few seconds, unable to keep eyes raised
Weakness worsened by pregnancy, hypokalaemia, infection, emotion, exercise and drugs - opiates, beta-blockers, gentamycin and tetracycline

229
Q

What is a possible differential for MG?

A

MS
Hyperthyroidism
Acute GBS
Lambert-Eaton myastheic syndrome
- Paraneoplastic condtion, most often seen with small cell lung cancer
- Causes defective ACh release at neuromuscular junction resulting in proximal limb weakness with some absent reflexes
- Weakness tends to improve after exercise unlike MG

230
Q

How is myasthenia gravis diagnosed?

A

Serum anti-AChR - raised in 90%, if negative then look for MuSK antibodies
Electromyography and nerve conduction study - EMG will detect MG, detect electrical activity from muscles, characteristic decrement occurs in evoked muscle action potential during repetitive stimulation
CT of thymus to look for hyperplasia, atrophy or tumour
Ptosis improves by < 2mm after ice application to shut affected lid for > 2 mins - nice non-invasive cheap test
Tensilon test - IV edrophonium given and muscle power increases within seconds - rarely used due to side effects

231
Q

How is MG treated?

A

Symptom control
- Anti-cholinesterase so more ACh remains in neuromusclar junction (also increased salivation, lacrimation, sweats, vomiting, miosis, diarrhoea)
Immunosuppression
- Used to treat relapses or if there is no response to pyridostigmine start low dose of steroids with dose increased per week - reduce dose on remission
- Steroids may be combined with oral azathioprine or methotrexate as disease becomes general
Thymectomy - removal of thymus in onset < 50 and disease poorly controlled with anti-cholinesterases

232
Q

What is the myasthenic crisis?

A

Weakness of respiratory muscle during relapse can be life-threatening
Monitor FVC
Treat with plasmapheresis and IV immunoglobulin and identify and treat trigger of relapse eg infection, medications

233
Q

What is a neuropathy?

A

Pathological process affecting a peripheral nerves

234
Q

What is a mononeuropathy?

A

Process affecting a single nerve

235
Q

What is mononeuritis multiplex?

A

Several individual nerves affected

236
Q

What is polyneuropathy?

A

Diffuse, symmetrical disease usually commencing peripheral
Can be motor, sensory, sensorimotor and autonomic
Classified into demyelinating and axonal types
Widespread loss of tendon reflexes typical, with distal weakness and distal sensory loss

237
Q

What are the 6 mechanisms that can cause nerve malfunction?

A
Demyelination
Axonal degeneration
Compression
Infarction
Infiltration
Wallerian degeneration
238
Q

What is demyelination?

A

Schwann cell damage leads to myelin sheath distribution

Results in marked slowing of conduction seen in GBS

239
Q

What is axonal degeneration?

A

Axonal damage causes nerve fibre to die back from periphery
Conduction velocity initially remains mortal because of axonal continuity maintained in surviving fibres
Axonal degeneration typically occurs in toxic neuropathies

240
Q

What is compression?

A

Focal demyelination at point of compression causes disruption of conduction
Typically occurs in entrapment neuropathies eg carpal tunnel syndrome

241
Q

What is infarction?

A

Micro-infarction of vasa nervorum occurs in diabetes and arteritis such as polyarteritis nodosa and eosinophilic gransulomatosis with polyangitis

242
Q

What is infiltration?

A

Occurs by inflammatory in leprosy and granulomas such as sarcoid and by neoplastic cells

243
Q

What is Wallerian degeneration?

A

Process that results when a nerve fibre is cut or crash and distal part of axon that is separated from neurones cell body degenerates

244
Q

What can cause mononeuritis multiplex?

A
WARDS PLC
Wegner's granulomatosis
AIDS/Amyloid
Rheumatoid arthritis
Diabetes mellitus
Sarcoidosis
Polyarteritis nodosa
Leprosy
Carcinoma
Carpal tunnel syndrome most common mononeuropathy
245
Q

What can cause a mononeuropathy?

A

Usually local causes such as trauma, entrapment (tumour)

246
Q

What is carpel tunnel syndrome?

A

Most common mononeuropathy and entrapment neuropathy

Results from pressure and compression on median nerve as it passes through the carpal tunnel in wrist

247
Q

When is carpal tunnel more common?

A

More common in women as they have narrower wrists but similar sized tendons to men

248
Q

What can cause carpal tunnel syndrome?

A
Idiopathic
Usually in those over 30
Associated with
- Hypothyroidism
- DM
- Pregnancy (third trimester)
- Amyloidosis including dialysis patients
- Obesity
- Rheumatoid arthritis
- Acromegaly
249
Q

How does carpal tunnel syndrome present?

A

Intermittent and gradual onset
Aching pain in hand and arm (especially at night) can wake patient up
Paraesthesiae in thumb, index, middle and half ring finger and palm
Relieved by dangling hand over the edge of the bed - wake and shake
May be sensory loss and weakness of abductor pollicis brevis +/- wasting of thenar eminence
Light touch, 2-point discrimination and sweating may be impaired

250
Q

What is a possible differential of carpal tunnel syndrome?

A

Peripheral neuropathy
Motor neurone disease
MS

251
Q

How is carpal tunnel syndrome diagnosed?

A

Electromyography
- See slowing of conduction velocity in median sensory nerves across carpal tunnel
- Prolongation of median distal motor latency
- Helps confirm lesion site and severity
Phalen’s test - can only maximally flex wrist for 1 minute
Tinel’s test - tapping on nerve at wrist induces tingling (non-specific)

252
Q

How do you treat carpal tunnel syndrome?

A

Wrist splint at night
Local steroid injection
Decompression surgery - carpal tunnel ligament cut to reduce pressure

253
Q

What are autonomic neuropathies?

A

Sympathetic or parasympathetic neuropathies may be isolated or part of generalised sensorimotor peripheral neuropathy

254
Q

What can cause autonomic neuropathies?

A
DM
GBS
Sjorgren's
HIV
SLE
255
Q

What are the symptoms of sympathetic neuropathies?

A

Postural hypotension - faints on standing, eating or hot bath
Ejaculatory failure
Reduced sweating

256
Q

What are the symptoms of parasympathetic neuropathies?

A

Erectile dysfunction
Constipation
Nocturnal diarrhoea
Urine retention

257
Q

What can cause cranial nerve lesions?

A
Tumour
MS
Trauma
Aneurysm
Vertebral artery dissection resulting in infarction
Infection - cerebellar abscess from ear
258
Q

What are the symptoms of causes of CN3 palsy?

A

Oculomotor
Ptosis
Fixed dilated pupil due to loss of parasympathetic outflow from Ediger-Westphal nucleus which supplies pupillary sphincter and ciliary bodies - lens accomodation
Eye down and out
Causes - raised ICP, diabetes, hypertension, giant cell arteritis

259
Q

What are the symptoms and causes of CN4 palsy?

A

Trochlear
Innervated superior oblique muscle and results in head tilt to correct extortion that results in diplopia and looking down
Caused by trauma to orbit

260
Q

What are the symptoms and causes of CN6 palsy?

A

Abducens
Adducted eye
Causes - MS, Wernicke’s encephalopthy, pontine stroke (presents with fixed small pupils +/- quadriparesis

261
Q

What are the symptoms and causes of a CN 3,4,6 palsy?

A

Non-function eye

Causes - stroke, tumours, Wernicke’s encephalopathy

262
Q

What are the symptoms and causes of CN5 palsy?

A

Trigeminal
Jaw deviated to side of lesion
Loss of corneal reflex
Causes - trigeminal neuralgia (pain but no sensory loss), herpes zoster, nasopharyngeal cancer

263
Q

What are the symptoms and causes of CN7 palsy?

A

Facial
Facial droop and weakness
Causes - Bells palsy most common lesion of facial nerve, fractures of petrous bone, middle ear infections, inflammation of parotid gland

264
Q

What are the symptoms and causes of CN8 palsy?

A

Vestibulocochlear
Hearing impairment
Vertigo and lack of balance
Causes - vestibulocochlear nerve runs very close to bone, very affected by surrounding tumours, skull fractures, toxic drug effects, ear infections

265
Q

What are the symptoms and causes of CN9 and CN10 palsy?

A
Glossopharyngeal and vagal
Gag reflex issues
Swallowing issues
Vocal issues
Caused by jugular foramen lesion
266
Q

How common are primary brain tumours?

A

Incidence of 8 per 100,000
16th most common adult cancer - disproportionate killer in young adults
2nd most common paediatric cancer
Account for less than 2% of all malignant tumours but 20% of childhood
In adults, majority supratentorial
In children - majority occur in posterior fossa
Majority are gliomas
- Astrocytoma (85-90%)
- Oligodendroglioma (5%)

267
Q

What can increase your risk of primary brain tumours?

A
More common in affluent groups
Ionising radiation
Vinyl chloride
Immunosuppression
FHx
268
Q

How are primary brain tumours graded?

A

Grade I - good prognosis, often completely benign
Grade II - > 5 year pronosis, premalignant tumour
Grade III - prognosis of 2-5 year survival, cancer, malignant, active growth and mitotic acitivty
Grade IV - < 1 year survival prognosis, most common phenotype, active growth, mitotic activity and necrosis, vascular proliferation, very malignant

269
Q

What are the pathways to malignant glioma?

A

Given enough time, all gliomas will progress to become glioblastoma multiforme except pliocytic astyocytoma (grade I)
Common pathway
- Especially in those under 50-60
- Initial genetic error of glucose glycolysis
- Mutation of isocitrate dehydrogenase I
- Results in excessive build-up of 2-hydroxyglutarate
- Triggers genetic instability in glial cells and subsequent inappropriate mitosis leading to cancer
Less common pathway
- More common in those over 50-60
- No IDH-1 mutation
- Resulting in very poor prognosis even for low grade tumours

270
Q

What is the pathology of oligodendromas?

A
Most common in 40s-50s
Arise from oligodendrocytes
Grow slowly, usually over several years
Calcification is common
May have seizures
WHO grade II
All are IDH-1 mutation positive
271
Q

Where do ependymomas come from?

A

Arise from ependymal cells

Line ventricles and spinal cord

272
Q

Where do meningiomas come from?

A

More common in older people and women

Benign and arise from arachnoid mater and may grow to a large size, usually over years, pushing into the brain

273
Q

Where do neurofibromas come from?

A

Solid benign tumours that arise from Schwann cell and occur principally in cerebellopontine angle

274
Q

Where do craniopharyngiomas come from?

A

Very rare benign tumours

Often diagnosed in younger patients

275
Q

How do tumours cause raised intracranial pressure?

A

Intracranial compartment is a complaint system
Initially when tumour is small, there will be no symptoms despite the rising ICP - due to brain compliance and ability to remove CSF from ventricle to spinal cord in order to offset the increase in pressure
Comes to a point where no more CSF can be relocated leading to a very sharp rise in ICP resulting in symptoms
Rising pressure results in midline shift of brain and herniation through foramen magnum

276
Q

How do primary brain tumours present?

A

Symptoms of raised ICP
- Progressive headache - worse on waking from sleep, pain so bad can wake up, increased by coughing, straining and bending forwards, sometimes relieved by vomiting
- Drowsiness
- +/- vomiting
- Papilloedema
Progressive neurological deficit depending on part of brain affected
- Temporal - dysphasia, amnesia
- Frontal lobe - hemiparesis, personality change, Broca’s dysphasia, lack of initiative, unable to plan tasks
Parietal lobe - hemisensory loss, reduction in 2-point discrimination, dysphasia, astereognosis
Occipital lobe - contralateral visual defects
Cerebellum - DASHING - Dysdiadochokinesis, Ataxia, Slurred speech, Hypotonia, Intention tremor, Nystagmus, GGait abnormality
Epilepsy/seizure of recent onset, partial/focal seizures more common
Lethargy/tiredness caused by pressure on brainstem

277
Q

What could be a differential of primary brain tumours?

A
Aneurysm
Abscess
Cyst
Haemorrhage
Idiopathic intracranial hypertension
278
Q

How are primary brain tumours diagnosed?

A

CT and MRI
- MRI superior for posterior fossa lesions
- Determine size and location
- High grade tumours have irregular edges and high growth rate
Bloods - FBC, U&Es, LFTs, B12
Biopsy
- Via skull burr-hole
- To determine cancer grade and confirm
Lumbar puncture contraindicated when possibility of mass lesion since withdrawing CSF my provoke immediate coning

279
Q

How are primary brain tumours treated?

A

Surgery to remove mass if possible
Chemotherapy for glioma - at same time as surgery than 6 weeks post-op, temozolomide, not all tumours sensitivie due to methyl-guanine-methyl-transferase
- If it has methylated MGMT then sensitive
Oral dexamethasone to reduce inflammation and oedema
Anti-convulsants

280
Q

How common are secondary brain tumours?

A
10x more common than primary brain tumours
Commonest neoplasms to metastasise to CNS in order
- Non-small cell lung
- Small cell lung
- Breast
- Melanoma
- Renal cell
- GI
281
Q

How are secondary brain tumours treated?

A

Surgery if < 75 years
Radiotherapy
Chemotherapy
Palliative therapy

282
Q

What is Huntington’s?

A

Cause of chorea

Neurodegenerative disorder characterised by lack of inhibitory neurotransmitter GABA

283
Q

What is chorea?

A

Continuous flow of jerky, semi-purposeful movements, flitting from one part of the body to another
They may interfere with voluntary movements but cease during sleep

284
Q

How common is Huntington’s?

A

Autosomal dominant condition with full penetrance
Incurable, progressive neurodegenerative disorder
Presents in middle age initially with a prodromal phase of mild symptoms eg irritability, depression and inco-ordination then progressing to psychiatric and cognitive symptoms
Very rare
Prevalence worldwide 5 per 100,000

285
Q

What causes Huntington’s?

A

Mutation on chromosome 4

Resulting in repeated expression of CAG sequence

286
Q

What can increase your risk of Huntington’s?

A

FHx

Having a parent with Huntington’s child has 50% risk of getting it

287
Q

What is the pathophysiology of Huntington’s?

A

Repeated expression of CAG sequence leads to translation of expanded polyglutamine repeated sequence in Huntington’s gene
Protein gene product function in unclear - expansion thought to be a toxic gain of function mutation
More CAG repeats present earlier symptom present
- Most adult onset have 36-55 repeats
- Early onset 60
Progressive cerebral atrophy with marked loss of neurones in caudate nucleus and putamen of basal ganglia - loss of gaba-nergic and cholinergic neurones
Results in decreased ACh and GABA synthesis in striatum
GABA main inhibitory neurotransmitter, loss of this will result in decreased inhibition of dopamine release therefore resulting in excessive thalamic stimulation and thus excessive movements

288
Q

How does Huntington’s present?

A

Prodromal phase of mild psychotic and behavioural symptoms
Chorea
- Relentlessly progressive, jerky, explosive, rigidity involuntary movements - ceases when sleeping
- Can’t sit still
- Begin as general restlessness, unintentionally initiated movements and lack of co-ordination
Dysarthria, dysphagia, abnormal eye movements
Psychiatric movements
- Behavioural change -aggression, addictive behaviour, apathy, self-neglect
- Depression/anxiety
Dementia
Associated with seizures
Eventually death - usually occurs within 15 years of diagnosis, usually from an intercurrent illness

289
Q

What is a differential of Huntington’s?

A

Other causes of chorea

  • Sydenham’s chorea (rheumatic fever)
  • Creutzfeldt-Jakob disease
  • Wilson’s disease
  • SLE
  • Stroke of basal ganglia
290
Q

How is Huntington’s chorea diagnosed?

A

Diagnosis mainly clinical
Genetic testing - many CAG repeats
Extensive genetic counselling required due to impact a positive diagnosis
CT/MRI showing caudate nucleus atrophy and increased size of frontal horns of lateral ventricles
- Imaging not useful early on

291
Q

How is Huntington’s treated?

A

No treatment to prevent progression
Counselling to patient and family, genetic counselling to children of patients
Symptomatic management of chorea
- Benzodiazepines, sulpiride (neuroleptic to depress nerve function), tetradenazine
Anti-depressants such as SSRIs
Antipsychotic medication such as neuroleptics
To treat aggression use risperidone

292
Q

What is depression?

A

Syndrome caused by a number of brain disorders which cause memory loss, difficulties with thinking, problem solving or languages as well as difficulties with activities of daily living
Alzheimer’s most common
Specific symptoms depend on parts of brain damaged and disease causing dementia

293
Q

How common is dementia?

A

Rare under 55
Prevalence rises with age
AD more common in women
Vascular and mixed dementia more common in men

294
Q

What are the different types of dementia?

A

Alzheimer’s disease (50%)
- Most common
- Degeneration of cerebral cortex, with cortical atrophy
- Accumulation of beta-amyloid peptide, degeneration product of amyloid precursor protein, results in progressive neuronal damage, neurofibrillary tangles, increases in number of amyloid plaques and loss of ACh
Vascular dementia (25%)
- Brain damage due to CVS disease
- Signs of vascular pathology
Lewy-body dementia (15%)
- Deposition of abnormal protein within neurons in brain stem and neocortex
- Presents with fluctuating cognitive impairment, details visual hallucinations
- Associated with Parkinson’s
Fronto-temporal dementia
- Specific degeneration or atrophy of frontal and temporal lobes of brain
- Behavioural and personality change, early preservation of episodic memory and spatial orientation, emotional unconcern, lowers inhibition
Mixed dementia
Parkinson’s dementia
Huntington’s
Liver failure
HIV
Prion disease
Vitamin B12/folate deficiency

295
Q

What can increase you risk of dementia?

A
FHx
Age
Down's syndrome
Alcohol use, obesity, high BP, hypercholesterolaemia
Diabetes
Atherosclerosis
Depression
High oestrogen levels
296
Q

What is Parkinson’s pathology overlap with dementia?

A

25% of all patients with AD with develop Parkinson’s

297
Q

How does AD present?

A

Insidious onset with steady progression over years
Short-term memory loss usually most prominent early symptoms, subsequently slow disintegration of personality and intellects
Eventually affects all aspects of cortical function
Decline in language, visuospatial skills, apraxia, agnosia

298
Q

How does vascular dementia present?

A

Stepwise deterioration with declines followed by periods of stability
History of TIAs and/or strokes
Evidence of atheropathy

299
Q

How does Lewy body dementia present?

A

Fluctuating cognition with pronounced variation in attention and alertness
Prominent or persistent memory loss may not occur in early stages
Impairment in attention, frontal, subcortical and visuospatial ability often prominent
Depression and sleep disorders occur
Visual hallucinations
Parkinson’s
Loss of inhibitions

300
Q

What is a differential of dementia?

A

Substance abuse
Hypothyroidism
Space-occupying intracranial lesions
Huntington’s

301
Q

How is dementia diagnosed?

A

Hx
MMSE - score of 25/30 normal, 18-24 mild-moderate, 17 or below
Exclusion of rare treatable causes of dementia should be considered
Blood tests - FBC, liver biochemistry, TFTs, vitamin B12/folate
Neuropsychology
Brain CT in younger patients or those with atypical presentation
MRI - extent of atrophy
Brain function assessed
- Energy and blood supply via PET and SPECT scanning
- Brain networks via functional MRI

302
Q

How is dementia treated?

A

No specific therapy
Support
- Socially active - talking to family and friends
- Cognitively acitve
- Specialist memory service
Carers
Medication
- Acetylcholinesterase inhibitor in Alzheimer’s to increased ACh
- BP control to reduce further vascular damage

303
Q

How is dementia prevented?

A

Health behaviours
Smoking cessation, good diet, physical activity and low alcohol
Engaging in more than 6 leisure activities
Education, occupation, premorbid IQ, mental acitivities decreases risk
Changes in CSF amyloid-beta seen 25 years before onset of symptoms

304
Q

What is trigeminal neuralgia?

A

Chronic, debilitating condition resulting in intense and extreme episodes of pain

305
Q

How common is trigeminal neuralgia?

A
Peak incidence between 50-60 years
More common in women
Prevalence increases with age
May be due to genetic disposition
Almost always unilateral
306
Q

What can cause trigeminal neuralgia?

A

Compression of trigeminal nerve by loop of vein or artery
Local pathology - aneurysms, meningeal inflammation, tumours
5th nerve lesion due to pathology
- Within brainstem - tumour, MS, infarction
- At cerebellopontine nagle - acoustic neuroma, another tumour
- Within petrous bone - spreading middle ear infection
- Within cavernous sinus - aneurysm of internal carotid, tumour or thrombosis of cavernous sinus

307
Q

What can increase your risk of trigeminal neuralgia?

A

Hypertension main risk factor

Triggers - washing affected area, shaving, eating, talking and dental prostheses

308
Q

What occurs in trigeminal neuralgia?

A

Compression of trigeminal nerve resulting in demyelination and excitation of nerve resulting in erratic pain signalling

309
Q

How does trigeminal neuralgia present?

A

Almost always unilateral
At least 3 attacks of unilateral facial pain
Facial pain occurs in one or more distributions of trigeminal nerve, with no radiation beyond trigeminal distribution
Pain has at least 3 of following
- Reoccuring in paroxysmal attacks from a fraction of a second to 2 minutes
- Severe intensity
- Electric shock like, shooting, stabbing or knife-like
- Precipitated by innocuous stimuli by affected side of face

310
Q

What is a differential for trigeminal neuralgia?

A

Giant cell arteritis
Temporal arteritis
Dental pathology
Temporomandibular joint dysfunction, migraine, cluster headaches

311
Q

How are trigeminal neuralgias diagnosed?

A

Needs to be at least 3 attacks with unilateral facial pain
Clinical diagnosis based on criteria above and based on Hx
MRI to exclude secondary causes or other pathologies
Not attributes to another disorder

312
Q

How are trigeminal neuralgias treated?

A
Typical analgesics and opioids do not work
Anticonvulsants
Less effective options
May spontaneously remit after 6-12 months
If drugs fail then surgery
- Microvascular decompression
- Gamma knife surgery
- Stereotactic radiosurgery
313
Q

What can cause median nerve lesions?

A

C6-T1
Wrist lesions - due to lacerations/carpal tunnel
Weakness of abductor pollicis brevis and sensory loss over radial 3.5 fingers and palm

314
Q

What are the symptoms and causes of ulnar nerve lesions?

A

C7-T1
Vulnerable to elbow trauma
Compression occurs at epicondylar groove or at point where nerve passes between 2 heads of flexor carpi ulnaris
Signs
- Weakness/wasting of medial wrist flexors, interossi, medial 2 lumbricals
- Wasting on hypothenar eminence, weak little finger abduction
- Sensory loss over medial 1.5 fingers and ulnar side of hand
- Flexion of 4th and 5ht DIP weak
- With lesion at wrist, claw hand more marked
Treatment
- Rest and avoiding pressure on nerve
- Night time soft elbow splint

315
Q

What are the causes and symptoms of radial nerve palsy?

A

C5-T1
Nerve opens to fist
Damaged by compression against humerus
Signs
- Test for wrist and finger drop with elvow flexed and arm pronated
- Sensory loss variable - dorsal of root of thumb most reliably affected
BEST
- Brachioradialis, extensors, supinator, triceps

316
Q

What are the causes and symptoms of brachial plexus palsy?

A

Pain/paraestheisa and weakness in affected arm in variable distribution
Causes
- Trauma, radiotherapy for breast carinoma, prolonged wearing of heavy rucksack, neuralgic amyotrophy, thoracic outlet compression

317
Q

How does phrenic nerve palsy prevent and what are the causes?

A

Orthopnoea with raised hemidiaphragm on CXR
Causes - lung cancer, myeloma, thymoma, cervical spondylosis, trauma, phrenic nucleus lesion, thoracic surgery, C3-5 zoster, HIV, muscular dystrophy

318
Q

How does lateral cutaneous nerve of thigh palsy present?

A

Meralgia paraesthetica
Antero-lateral burning thigh pain from entrapment under inguinal ligament
L2-3

319
Q

How does sciatic nerve palsy present?

A

L4-S3
Damaged by pelvis tumours or fractures to pelvis or femur
Lesions affect hamstrings and all muscles below knee
Foot drop
Loss of sensation below knee laterally

320
Q

How does common peroneal nerve lesion present?

A

L4-S1
Originates from sciatic nerve just above knee
Often damaged as it winds around fibular head due to trauma or sitting cross legged
Foot drop
Weak ankle dorsiflexion
Eversion
Sensory loss over dorsum of foot

321
Q

How does tibial nerve lesion present?

A

L4-S3
Originates from sciatic nerve just above knee
Lesions lead to an inability to stand on tiptoe, invert foot or flex toes
Sensory loss over sole

322
Q

How should you examine a head injury?

A
GCS up to 14 score
- Motor response
- Verbal response
- Eye opening response
Inaccurate within 1 hour of event
Lateralising signs - identifies which hemisphere issue is in, check painful stimuli response
Look for pupils for signs of raised ICP
323
Q

How do you manage a head injury?

A
IV mannitol (diuretic to reduce oedema)
Management of seizures 
Monitor vital signs
Intubation if GCS < 8
Neurosurgery - ICP monitor insertion, Burrholes or craniotomy to relieve ICP
Suture scalp lacerations before CT
324
Q

How are cerebellar diseases classified?

A
Congenital ataxias
Diseases where ataxia is one of many features
Episodic ataxia
Autosomal recessive ataxia
- Friedreich's ataxia
Autosomal dominant ataxia
- Spinocerebellar ataxia 6
Sporadic ataxia
- Gluten ataxia
- Toxic effects
325
Q

Which area of cerebellum is most important?

A

Purkinje layer as only output

Results in dysfunction and ataxia

326
Q

How does cerebellar disease present?

A
Dysarthria
Dysphagia
Clumsiness
Intention tremor
Unsteadiness when walking
Stumbling and falls
Cognitive problems
Nystagmus
Limb and gait ataxia
327
Q

How is cerebellar disease diagnosed?

A

MRI to look for

  • Cerebrovascular damage
  • Primary tumours
  • Secondary tumours
  • Hydrocephalus
  • MS
328
Q

What is Brown-Sequard syndrome?

A

Hemi-section of spinal cord

Ipsilateral and contralateral are in relation to lesion

329
Q

How does Brown-Sequard syndrome present?

A

Ipsilateral weakness below lesion - damage of ipsilateral descending motor tracts
Ipsilateral loss of dorsal column proprioception below lesion - ascending tracts damaged before they could decussate
Contralateral loss of spinothalamic pain and temperature

330
Q

How does tramadol work?

A

Opioid effects and some serotonin reuptake inhibition

Class 3a controlled drug

331
Q

Give an example of an SSRI

A

Citalopram
Fluoxetine
Sertraline

332
Q

What are SSRIs used for?

A

First line treatment for moderate to severe depression and in mild depression if other treatments fail
Panic disorder or OCD

333
Q

How do SSRIs work?

A

Preferentially inhabit neuronal reuptake of serotonin from synaptic cleft so increase availability for neurotransmission
Improves mood and physical symptoms of depression and relieves symptoms of panic

334
Q

What are the main side effects of SSRIs?

A

GI disturbance, appetite, weight disturbance, hypersensitivity reactions
Hyponatraemia
Suicidal thoughts/behaviours may increase
Lowers seizure thresholg
Prolonged QT
Use with care in people with peptic ulcers, epilepsy, hepatic impairment, may not be effective in young people
CI with other monoamine oxidase inhibitors as may cause serotonin syndrome
CI with other drugs that prolong QT
May also increase bleeding when used with anticoagulants

335
Q

Name an example of a tricyclic antidepressant

A

Amitriptyline

336
Q

What are tricyclic antidepressants used for?

A

Second line treatment for moderate to severe depression where SSRIs are ineffective

337
Q

How do tricyclic antidepressants work?

A

Inhibit neuronal uptake of serotonin and noradrenaline from synaptic cleft so increase availability
Improves mood and physical symptoms
Blocks wide range of receptors accounting for range of side effects

338
Q

What are the main adverse effects of tricyclic antidepressants?

A

Dry mouth, constipation, urinary retention, blurred vision, sedation, hypotension, arrhythmias, QT and QRS prolongation, hallucinations and sexual dysfunction
Use with caution in people particularly at risk of adverse effects ie elderly, people with CV disease or epilepsy
CI with monoamine oxidase inhibitors as serotonin levels can get too high

339
Q

Name an example of a benzodiazepine

A

Diazepam

Lorazepam

340
Q

What are benzodiazepines used for?

A

First line treatment of seizures and status epilepticus
Management of alcohol withdrawal reactions
Short term treatment of severe anxiety or insomnia

341
Q

How do benzodiazepines work?

A

Target GABA-a receptor which is a chloride channel that opens in response to binding GABA opening channel allows chloride to enter making cell more resistant to depolarisation
Facilitate enhanced binding of GABA to GABA-a receptors and have widespread depressant effect on synaptic transmission
Can thus cause, reduced anxiety, sleepiness, sedation, and anti-convulsion

342
Q

What are the main adverse effects of benzodiazepines?

A

Cause dose dependent drowsiness, sedation, coma and in overdose loss of airway reflexes which can lead to coma and death
Use in elderly, those with respiratory or hepatic impairment and neuromuscular disease with care
Increased sedative effects when used with other drugs such as alcohol, opioids
With cytochrome P450 inhibitors may increase their effects

343
Q

What is carbamezepine?

A

An anticonvulsant

344
Q

What is carbamazepine used for?

A

First line treatment for epilepsy - specifically partial/focal seizures with/without secondary generalisation and for primary generalised seizures
Trigeminal neuralgia to control pain and reduce attack severity

345
Q

How does carbamazepine work?

A

Incompletely understood - inhibits neuronal Na+ channels, stabilising resting membrane potential and reducing neuronal excitability
May inhibit spread of seizure acitivty in epilepsy and control neuralgic pain by blocking synaptic transmission in trigeminal nucleus

346
Q

What are the main adverse effects of carbemazipine?

A

GI disturbance, dizziness, ataxia, hypersensitivity skin rash, oedema, hyponatraemia
CI in prior anti-epileptic hypersensitivity syndrome
Used with caution in hepatic, renal and cardiac disease
Reduces efficacy of drugs metabolised by P450 enzymes eg warfarin, oestrogens, progesterones
Effects increased by cytochrome P450 inhibitors
Efficacy reduced by drugs that lower seizure threshold eg SSRIs

347
Q

Name 2 other anticonvulsants

A

Gabapentin

Pregabalin

348
Q

What are gabapentin and pregabalin used for?

A

Partial/focal epilepsies, usually as add on when other drugs such as carbamazepine insufficient
Also used for neuropathic pain, mirgraine prophylaxis, and generalised anxiety disorder

349
Q

How does gabapentin work?

A

Closely related to GABA
Binds to voltage sensitive Ca2+ channels, where is prevents inflow of Ca2+ and thus inhibits neurotransmitter release - interfering with synaptic transmission and reducing neuronal excitability

350
Q

What are the main adverse effects of gabapentin?

A

Generally better tolerated
Drowsiness, dizziness, ataxia
Use with care in renal impairment
Sedative effects enhanced when used with other sedating drugs

351
Q

Name a dopaminergic drug

A

Levodopa

352
Q

What is levodopa used for?

A

Parkinson’s

353
Q

How does levodopa work?

A

Increases dopaminergic stimulation to basal ganglia
Dopamine itself cannot cross BBB but levodopa as a dopamine precursor can
Once in brain converted to dopamine in dopaminegic neurones

354
Q

What are the main adverse effects of levodopa?

A

Nausea, drowsiness, confusion, hallucinations, hypertension
Major issue - effects wear off towards end of dosage intervals
Increasing dose can lead to excessive movement causing on/off effect
Use with care in elderly and those with existing cognitive disease
Use with caution in CVS disease
Always give with decarboxylase inhibitor such as co-careldopa to reduce levodopas peripheral conversion before it is able to enter the brain - reduces nausea and lowers dose required
Do not use with antipsychotics as they contraindicate effects of levodopa

355
Q

Name a dopamine depleting drug

A

Reserpine

356
Q

What are dopamine depleting drugs used for?

A

Treating dyskinesia in Huntington’s disease

Used as antipsychotic and antihypertensive

357
Q

How do dopamine depleting drugs work?

A

Monoamine neurotransmitters are dopamine, serotonin, noradrenaline and adrenaline
Irreversibly blocks vesicular monoamine transporter which normally transports free intracellular noradrenaline, serotonin and dopamine for release across synapse
Post synaptic cells not activated and symptoms decrease - effects long lasting until VMAT replenished

358
Q

What are the main adverse effects of dopamine depleting drugs?

A
Nasal congestion
GI disturbance
Drowsiness/dizziness
Hypotension and bradycardia so use with care in patients with CVS disease
May also worsen asthma
359
Q

Name an example of a triptan

A

Sumatriptan

Rizatriptan

360
Q

What are triptans used for?

A

Treatment of migraines and cluster headaches

361
Q

How do triptans work?

A

Agonist of serotonin 5-HT-1b and 1d receptors in cranial blood vessels causing constriction and subsequent inhibition of pro-inflammatory neuropeptide release
May also work on serotonin receptors in nerve endings to relieve pain through decrease in levels of substance P

362
Q

What are the main adverse effects of triptans?

A

Recurrence of migraine - more than without drug
Rare risk of coronary spasm and cardiac events
CI in previous/current severe CVD or stroke
Use with caution in older people and FHx of stroke, CVD, diabetes and high cholesterol

363
Q

Name an example of a monoamine oxidase inhibitor

A

Selegiline

Rasagiline

364
Q

What are monoamine oxidase inhibitors used for?

A

Parkinson’s
Alzheimer’s
Atypical depression

365
Q

How do monoamine oxidase inhibitors work?

A

Inhibits activity of monoamine oxidase preventing breakdown of monoamine neurotransmitters (dopamine, serotonin, adrenaline, noradrenaline) and increasing their availability
MAO-B preferentially targeted in Parkinson’s and Alzheimer’s disease as found to be increased

366
Q

What are the main adverse effects of monoamine oxidase inhibitors?

A

Hypertensive crisis if large amounts of tyramine are consumed so cut down on aged cheeses and alcohol
Should not be used with other psychoactive drugs such as antidepressants and adrenaline

367
Q

Name an example of an acetlycholinesterase inhibitor?

A

Rivastigmine

Donepezil

368
Q

What are acetylcholinesterase inhibitors used for?

A

Alzheimer’s
Lewy body dementia
Parkinson’s
Myasthenia gravis

369
Q

How do acetylcholinesterase inhibitors work?

A

Inhibit breakdown of acetylcholine by blocking site of acetylcholinesterase
In neurodegenerative disorders used to treat memory and learning deficit symptoms which improve due to role of acetylcholine in cognition
In myasthenia gravis too few acetylcholine receptors so with acetylcholinesterase blocked and acetylcholine transmission increased, then acetlycholine can bind to few receptors more easily and trigger more muscular contractions

370
Q

What are the main adverse effects of acetylcholinesterase inhibitors?

A

Actions on parasympathetic nervous system can cause bradycardia, hypotension, hypersecretion, bronchoconstriction
GI tract hypermotility
Prolonged muscular contraction
CI in urinary retention due to obstruction
Use with care in people with CVS disease ie arrhythmias and in asthmatics